1. Which of the following pairs is/are highlands of in correctly matched? , and . Crops Feature Karnataka alone accounts for more than in two third of total production of coffee in the country. 1 Wheat : It requires 210 Pair 4 is correct. Tea is a plantation . frost-free days. crop. It is grown over undulating 2 Cotton : It requires high topography of hilly areas and well . temperature and drained soils in humid and sub-humid bright sunshine. tropics. Tea is cultivated on the lower 3 Coffee : It is cultivated in slopes of Nilgiri and . . the highlands. Source: Vajiram and Ravi - Yellow 4 Tea : It is cultivated on Book, Indian Geography, Chapter 9: . the lower slopes Agriculture, Page no – 188 of Nilgiri hills. Select the correct answer using the 2. As per Census 2011 of India, code given below: highest out-migration of people is (a) 1 only witnessed from which among the (b) 2 and 4 only following states of India? (c) 2, 3 and 4 only (1) (d) 1, 2, 3 and 4 (2) (3) Uttar Pradesh Answer: (c) Select the correct answer using the Explanation: code given below: Pair 1 is incorrect. 210 frost-free days (a) 1 only are required for Cotton crop and not (b) 2 only Wheat. Wheat is primarily a crop of (c) 1 and 3 only temperate zone. Its cultivation in India is (d) 2 and 3 only done during winter. Hence, it is a rabi crop. Around 85 per cent of the total area Answer: (c) under this crop is concentrated in north Explanation: and central regions of the country. Uttar Migration is an important factor affecting Pradesh, Punjab, Haryana, Rajasthan the course of socio-economic and Madhya Pradesh are five leading development in India. Accelerated wheat producing states in India. movement of people originating mainly Pair 2 is correct. Cotton requires high from the rural and backward areas in temperature, light rainfall or irrigation, search of employment has been one of 210 frost-free days and bright sunshine the most important features of the labour for its growth. It is a Kharif crop and market scenario in India during the post- requires 6 to 8 months to mature. It Independence period. grows well in drier parts of the black According to the World Economic cotton soil of the Deccan plateau. Major Forum's report on 'Migration and Cities', cotton-producing states are – interstate migration in India doubled , Gujarat, Madhya Pradesh, between 2001 and 2011 compared to the etc. previous decade. Census 2011 pegs the Pair 3 is correct. Coffee is a tropical total number of internal migrants in the plantation crop. Coffee is cultivated in the country (accounting for inter- and intra-

Prelim IAS Test Series (2019) – GS Test 12 (24.02.2019) 1 Geography 2 and Current Affairs Sep, Oct, Nov, Dec 2018 state movement) at a staggering 139 Statement 2 is correct: The Peninsular million. Uttar Pradesh and Bihar are the plateau consists of mostly relicts and biggest source states, followed closely by residual mountains like the Aravalli hills, Madhya Pradesh, Punjab, Rajasthan, the Nallamala hills, the Javadi hills, the Uttarakhand, Jammu and Kashmir and Veliconda hills, the Palkonda range, the West Bengal. The major destination Mahendragiri hills, etc. The plateau has states are Delhi, Maharashtra, Tamil senile topographic features like small Nadu, Gujarat, Andhra Pradesh and rounded hills due to prolonged erosion. Kerala. Source: Vajiram and Ravi Yellow Source: Vajiram and Ravi - Yellow Book, Indian Geography, Chapter 2: Book, Indian Geography, Chapter 15: Physiography of India; Page no. 26 Urbanization in India, Page no: 346 4. Consider the following map: 3. Consider the following statements with reference to the 'Indian Peninsular Plateau': (1) It supports less population than that of the northern plains. (2) It consists of mostly relicts and residual mountains. Which of the statements given above is/are correct? (a) 1 only (b) 2 only (c) Both 1 and 2 (d) Neither 1 nor 2

Answer: (c) Explanation: The Indian Peninsular Plateau lies to the south of the Indo-Gangetic plain and is Which of the area(s) marked in the flanked by the sea/ocean on three sides. map above (with X, Z and O The Peninsula is formed essentially by a letters) represent arid and semi- great complex of very ancient gneisses arid climatic zones of India? and granites, which constitutes a major (a) X only part of it. (b) X and O only Statement 1 is correct: The Peninsular (c) Z and O only plateau supports less population than (d) X and Z only that of northern plain because of its rugged terrain and water scarcity. The Answer: (d) northern plain of India is densely Explanation: populated because it is a large plain of The areas marked with ‘X’ and ‘Z’ letters alluvial soil. The deposition of alluvium in represent the hot ‘arid and semi arid’ a vast basin lying at the foothills of the climatic zones respectively. The area makes the northern plain very marked as ‘O’ represents tropical wet fertile. regions of India.

Prelim IAS Test Series (2019) – GS Test 12 (24.02.2019) 2 Geography 2 and Current Affairs Sep, Oct, Nov, Dec 2018 The term “arid” normally means a region of the earth’s surface where rainfall is nil or inadequate. Hence, the vegetation is non-existent or sparse, agriculture difficult or impossible and human living conditions precarious. Generally, arid regions are those that receive less than 25 cm annual rainfall. It includes all hot and cold deserts. These regions are devoid of any significant vegetation due to moisture stress. Regions receiving annual rainfall between 25 and 75 cm can be termed as semi arid regions.  The hot Indian arid zone lies in the north-west part of the India and comprise largely of the desert region of Rajasthan. The description given above is  Semi-arid zones occur chiefly in the most suitable for which one of the states of Rajasthan, Punjab, following: Haryana, Uttar Pradesh, Gujarat, (a) Wild Ass Madhya Pradesh, Maharashtra, (b) Sangai Karnataka, Andhra Pradesh and (c) Nilgai Tamil Nadu as shown in the map (d) Leopard given above. Source: NCERT – Class XI, India – Answer: (a) Physical Environment, Chapter: Explanation: Structure and Physiography, Chapter Indian Wild Ass is one of the sturdiest – 2, Page no. 17 animals in the world withstanding scorching mid-day temperature up to 48° 5. Consider the following features C or more without any shelter. It has the about a particular ‘Indian Wildlife strength and speed of a horse and can species’: run at the speed of 60-70 km/hr. The wild (1) It is found in the Little Rann ass, locally known as ghudkhar and of Kutch in Gujarat and its found only in the Little Rann of Kutch in surrounding areas. Gujarat in India, has been classified as (2) It is protected under an endangered animal. It is protected Schedule I of the Wildlife under Schedule I of the Wildlife (Protection) Act 1972. (Protection) Act 1972. It is included in (3) It is included in Appendix I of Appendix I of the Convention on the Convention on International Trade in Endangered International Trade in Species (CITES), making international Endangered Species. trade in this species illegal. Source: Vajiram and Ravi - Yellow Book, Indian Geography Chapter 6: Natural Vegetation and Wildlife, Page no. 106

Prelim IAS Test Series (2019) – GS Test 12 (24.02.2019) 3 Geography 2 and Current Affairs Sep, Oct, Nov, Dec 2018 6. Select the correct order of the 7. With reference to the soils found in meeting points (from East to West) India, consider the following pairs: of the following Ganga tributaries Soil Region where they join the main stream of 1 Alluvial : Indo- river Ganga? Soils Gangetic (1) Gomati Plains (2) Gandak 2 Laterite : Great Indian (3) Yamuna Soils Desert (4) Kosi 3 Black : Deccan Trap (5) Soils Select the correct answer using the Which of the pairs given above code given below: is/are correctly matched? (a) 3-1-5-4-2 (a) 2 only (b) 3-4-5-1-2 (b) 1 and 2 only (c) 4-2-5-1-3 (c) 1 and 3 only (d) 4-2-1-5-3 (d) 1, 2 and 3

Answer: (c) Answer: (c) Explanation: Explanation: The correct order of the meeting points Pair 1 is correct. Alluvial Soils are the (from East to West) of Ganga tributaries most common soils in India. These are where they join main stream of river widespread in northern plains and river Ganga is: Kosi - Gandak - Ghaghara - valleys. These are rich in humus, lime Gomati - Yamuna. and organic matter. These are highly The Ganga is the most important river of fertile. India both from the point of view of its Pair 2 is incorrect. The name of laterite basin and cultural significance. It rises in soils is derived from Latin word ‘Later’ the Gangotri glacier near Gaumukh which means a Brick. These soils are (3,900 m) in the Uttarkashi district of most common in the areas of high Uttaranchal. Here, it is known as the temperature and high rainfall and are Bhagirathi. It cuts through the Central formed due to leaching. In India, laterite and the Lesser Himalayas in narrow soils are found on the slopes of the gorges. The Son is its major right bank Western Ghats and not in Indian desert. tributary. The important left bank Pair 3 is correct. Black Soils are most tributaries are the Ramganga, the common in Deccan Trap. These are Gomati, the Ghaghara, the Gandak, the mature soils with high water retaining Kosi and the Mahanada. The river finally capacity. These soils are ideal for discharges itself into the cultivation of cotton. near the Sagar Island. Source: NCERT Class XI, India: Note: The Yamuna, is the western most Physical Environment, Chapter 6, and the longest tributary of the Ganga. Page no 68-71. Source: NCERT class XI, India Physical Environment, Chapter – Drainage system, Chapter - 3, Page – 22

Prelim IAS Test Series (2019) – GS Test 12 (24.02.2019) 4 Geography 2 and Current Affairs Sep, Oct, Nov, Dec 2018 Andhra Pradesh has the second longest mainland coastline with a total length of 974 km. Statement 2 is correct: Coastal area covers more than 10% of the earth surface. Because of the economic benefits that accrue from access to ocean navigation, coastal fisheries, tourism, recreation and industrialization, human settlements are often more concentrated in the coastal zone than elsewhere. About 40% of the world’s population lives within 100 km of the coast. About 35% of Indians live within 100 km of the country’s coastline measuring 7517 km. Source: NCERT – Class XI, India – Physical Environment, Chapter – Structure and Physiography, Chapter no. 2, Page no. 18

9. A man residing in southern India 8. Consider the following statements: visited Sarnath for his spiritual trip. (1) In India, Andhra Pradesh has He travelled mostly on a particular the longest mainland National Highway. Some details coastline. about his journey are mentioned (2) The coastal plains are one of below: the most densely populated (1) He started from areas of the world. southernmost point of Which of the statements given peninsular India. above is/are correct? (2) He got a chance to see (a) 1 only Silicon Valley of India on the (b) 2 only way. (c) Both 1 and 2 (3) He encountered 'Tiger (d) Neither 1 nor 2 Capital of India' during his journey. Answer: (b) If these things occurred in a Explanation: sequence as mentioned above The total length of India’s coastline is then the man must have used 7516 km. Of this, mainland coastline which one of the following National length is 5422 km whereas Island Highways? territories coastline length is 2094 km. (a) National Highway 5 Statement 1 is incorrect: The 1214.7 (b) National Highway 6 km long coastline of Gujarat, which is (c) National Highway 7 approximately 23% of the country’s total (d) National Highway 8 mainland coastline, makes it the state with the longest mainland coastline. Answer: (c)

Prelim IAS Test Series (2019) – GS Test 12 (24.02.2019) 5 Geography 2 and Current Affairs Sep, Oct, Nov, Dec 2018 Explanation: National Highway 7 is renamed as National Highway 44, The 2,369 km long road is the longest National Highway of India and one of the major road network between south and north India. It connects Varanasi to Kanyakumari. Sarnath is 10km from Varanasi. The National Highway 7 or 44 connects major Indian cities like Varanasi, Nagpur, Hyderabad, Bangalore, Madurai, Kanyakumari. Kanyakumari is the southernmost point of peninsular India. It is the meeting point of three seas/oceans-the Bay of Bengal, Which of the following crops are the Arabian Sea and the Indian Ocean. cultivated under the areas as The Silicon Valley of India is a nickname marked (with X and O letters for the Indian city of Bangalore. The respectively) in the India map given name signifies Bangalore’s status as a above? hub for information technology (IT) (a) Coffee, Bajra companies in India. (b) Tea, Wheat Nagpur is called as the Tiger Capital of (c) Jute, Groundnut India. Maharashtra has done (d) Coffee, Jute exceptionally well in the growth of tigers - which has prompted government to Answer: (d) promote Nagpur as the “Tiger Getaway” Explanation: of India. Coffee: Commercial plantations of coffee Value Addition: started in India during the 18th century.  National Highway 8 - From Delhi to Coffee grows in well drained soil, tropical Dankuni temperatures, well distributed rains and  National Highway 6 - From Hajira to shade (as sunshine breaks open the Kolkata beans). Hence, the area under coffee  National Highway 5 - From Cuttack to plantations in India is concentrated in the Chennai southern states of Karnataka (53.83%), Source: Vajiram and Ravi - Yellow Kerala (18.89%) and Tamil Nadu Book, Indian Geography Chapter 13: (7.83%). Transport and Communication Jute: Jute is soft, flexible and hard- Network, Page no. 279 wearing fibre. Jute fibres are long, lustrous, resilient and softer to touch. 10. Consider the following map: Jute crop requires humid climate with temperature fluctuating between 24 degree Celsius and 38 degree Celsius. Minimum rainfall required for jute cultivation is 100 cm. New grey alluvial soil of good depth receiving silt from annual floods is most suitable for jute

Prelim IAS Test Series (2019) – GS Test 12 (24.02.2019) 6 Geography 2 and Current Affairs Sep, Oct, Nov, Dec 2018 growth. The cultivation of jute in India is Statement 2 is correct. Per capita mainly confined to the eastern region of consumption of electricity is also another the country. West Bengal alone accounts important indicator of regional disparities. for over 50 percent raw jute production. States like Punjab, Gujarat, Haryana, The leading world's jute producing Maharashtra etc., having higher degree countries are India, Bangladesh, China of industrialisation and mechanisation of and Thailand. India is the world's largest agriculture, have recorded a higher per producer of raw jute and jute goods, capita consumption of electricity than the contributing to over 50 percent and 40 economically backward states like percent respectively of global production. Assam, Bihar, Odisha, Madhya Pradesh Source: NCERT XII - India, India and Uttar Pradesh. People and Economy, Chapter 5: Land Source: Vajiram and Ravi - Yellow Resources and Agriculture, Page 53- Book, Indian Geography, Chapter 16: 54. Regional development and Planning, Page no. 362 11. In the context of Indian states, consider the following statements: 12. Arrange the following features in (1) Per capita income of is the order of their occurrence (from relatively more than that of north to south) on the India Map: Bihar. (1) (2) Per capita consumption of (2) electricity is relatively higher (3) in Gujarat than in Odisha. (4) Palghat Gap Which of the statements given Select the correct answer using the above correctly indicates the code given below: regional disparity in India? (a) 1-2-4-3 (a) 1 only (b) 1-4-2-3 (b) 2 only (c) 3-2-1-4 (c) Both 1 and 2 (d) 3-1-2-4 (d) Neither 1 nor 2 Answer: (d) Answer: (c) Explanation: Explanation: Zoji La is a high mountain pass in the Both the statements (1) and (2) correctly Indian state of Jammu and Kashmir, indicate the regional disparity in India. located on National Highway 1 between Statement 1 is correct. The most Srinagar and Leh in the western section important indicator of regional disparity of the Himalayan mountain range. It among different states of India is the separates Kashmir Valley from Dras difference in per capita state income Valley. figures. As per the report of the Ministry Rohtang Pass is a high mountain pass of Statistics and Programme at an elevation of 3979 m above the sea Implementation (released on 20 Aug level, located on the eastern Pir Panjal 2015), the state per capita income of Range of the Himalayas. The pass Goa (Rs. 2,42,745) is highest in the connects Kullu valley with Lahaul and country and of Bihar (Rs. 31,380) is Spiti which in turn provide access to Leh. lowest in the country.

Prelim IAS Test Series (2019) – GS Test 12 (24.02.2019) 7 Geography 2 and Current Affairs Sep, Oct, Nov, Dec 2018 Bhor Ghat is located near in Maharashtra. The Mumbai Pune highway passes through this gap. Gap or Palghat Gap is a low mountain pass in the Western Ghats between in Tamil Nadu and Palakkad in Kerala. It has an average elevation of 140 metres. The pass is located between the Nilgiri Hills to the north and to the south. Source: NCERT Class 11th, India: Physical Environment, Chapter 2: Structure and Physiography, Page 17.

13. With reference to ‘Wastelands in India’, consider the following statements: can result from inherent disabilities such (1) Wastelands can be formed as location, environment, chemical and due to erosion by water but physical properties, etc. They even suffer not due to anthropogenic from imposed disabilities like factors. management conditions. (2) Rajasthan has highest area Statement 1 is incorrect. Wastelands under wasteland in India can be formed due to erosion by water. whereas Delhi has the least This includes sheet, rill and gully erosion. area. Also, anthropogenic factors like mining, Which of the statements given shifting cultivation, etc contribute to the above is/are correct? formation of wastelands. (a) 1 only Statement 2 is correct. Rajasthan tops (b) 2 only the chart with 25% of its geographical (c) Both 1 and 2 area being wasteland, accounting for (d) Neither 1 nor 2 18% of the wasteland nationally. Maharashtra, Madhya Pradesh, Andhra Answer: (b) Pradesh, Himachal Pradesh have Explanation: significant percentage of wasteland. Delhi stands at the bottom of the pile with not an inch of wasteland. Source: NCERT Class 12th, India: People and Economy, Chapter 5: Land Resources and Agriculture, Page 41.

14. Which among the following factors can influence the growth of Indian population? (1) Total Fertility Rate Wastelands are the lands which due to (2) Migration neglect or degradation, are not being (3) Literacy rate utilized to their full potential. Such lands

Prelim IAS Test Series (2019) – GS Test 12 (24.02.2019) 8 Geography 2 and Current Affairs Sep, Oct, Nov, Dec 2018 Select the correct answer using the code given below: 15. Which of the following is the (a) 1 and 2 only primary reason behind large scale (b) 2 and 3 only ‘Coral Bleaching,’ recently (c) 1 and 3 only witnessed off the coast of (d) 1, 2 and 3 Andaman and Nicobar Islands? (a) Aggressive fishing techniques Answer: (d) (b) Increase in siltation Explanation: (c) Discharge of chemicals Point 1 is correct. India’s total fertility (d) Rise in sea-surface rate - a measure of the number of temperature children born to a woman during her lifetime - was down from 5.9 in 1951 to Answer: (d) 2.3 in 2011. It was 3.2 at the time when Explanation: National Population Policy, 2000 was Corals support immense biodiversity as adopted. India is expected to reach around 25% of the world’s fish species replacement level fertility of 2.1 by 2020. are dependent on them. Corals can grow So, India is actually close to stabilizing its under narrow environmental conditions - population. temperature between 22 and 26°C, Point 2 is correct. In countries like India, salinity between 32 and 38 parts per immigration plays a very small role in the thousand with more than 90% water population change. Although people from clarity. neighbouring countries like Bangladesh, Coral bleaching, whitening of coral that Pakistan and migrate to India; at results from the loss of a coral’s the same time Indians migrate to other symbiotic algae (zooxanthellae) or the countries like the US, Australia, and the degradation of the algae’s photosynthetic UK. During the 1971 war between India pigment. Such bleaching is associated and Pakistan over Bangladesh, the with the devastation of coral reefs. immigration rate increased tremendously. In 2016, there was a loss of more than Point 3 is correct. Literacy rate in India 23% corals off the coast of Andaman and is defined as the total percentage of the Nicobar Islands when the sea-surface population in the age group of seven temperature increased due to El-Nino years or above which can read and write effect. In a single year, the reef cover with understanding. Compared to barely went down from 52.27% to 39.94% in 18 percent of India’s population recorded India. as literate in the first Census after Note: El-Niño effect is the irregular Independence, according to the 2011 periodic warming of the eastern Pacific Census, that proportion has gone up to Ocean that affects the climate in the 74 percent. Compared with illiterate tropical and sub-tropical regions. young women, educated ones desire Source: NCERT XI- India: Physical smaller families and generally manage to Environment, Chapter 4: Climate, achieve that goal. Better access to family Page no-38-53 planning and contraception must go hand-in-hand with improved education. 16. The Narmada valley lies between Source: Vajiram and Ravi - Yellow which of the following mountain Book, Indian Geography, Chapter 14: ranges? Population, Page no: 306 (a) Bhander and Maikal

Prelim IAS Test Series (2019) – GS Test 12 (24.02.2019) 9 Geography 2 and Current Affairs Sep, Oct, Nov, Dec 2018 (b) Satpura and Aravalli Source: Vajiram and Ravi - Yellow (c) Satpura and Vindhyas Book, Indian Geography, Chapter 3 – (d) Vindhyas and Aravalli Drainage System and River Water, Chapter 3, Page no 44 Answer: (c) Explanation: 17. India is rich in a few minerals but The Satpura and Vindhaya ranges run lacks some other important ones. parallel to each other. The Satpura In this context, consider the Range is a mountain range that stretches following statements: through the central part of India. It (1) India is the largest coal extends across Maharashtra, Madhya producer in the world. Pradesh and Chhattisgarh. The Vindhya (2) India is the second largest Range geographically separates producer of diamonds after Northern India from Southern India as it Australia. runs across central India. It is a very old Select the correct answer using the mountain range in Central India. It code given below: extends in east-west direction from (a) 1 only Varanasi to Gujarat through Madhya (b) 2 only (c) Both 1 and 2 (d) Neither 1 nor 2

Answer: (d) Explanation: On the basis of chemical and physical properties, minerals are grouped as − Metallic minerals and Non-metallic minerals. Major examples of metallic minerals are iron ore, copper, gold, etc. Depending upon the origination, non- metallic minerals are either organic (such as fossil fuels also known as mineral Pradesh. fuels, which are derived from the buried Most of the major Peninsular Rivers animal and plant, e.g. such as coal and except Narmada and Tapi flow from west petroleum), or inorganic minerals, such to east. Narmada, the largest west as mica, limestone, graphite, etc. flowing river of the peninsula, rises near Statement 1 is incorrect. India is 3rd Amarkantak range in Madhya Pradesh. It largest producer of coal, 4th largest iron is the longest river of Indian Peninsula ore producer, and 3rd largest steel which drains into Arabian Sea. It is one of producer. It has 7th largest reserves of the rivers in India that flows in the rift Bauxite. valley, between Satpura and Vindhyas Statement 2 is incorrect. India is poor range. The Narmada flows in trough in Diamond deposits. (Panna diamond faults and fill the original cracks with their belt, Madhya Pradesh is one of the few detritus materials. Hence, there is a lack areas where diamond is found). of alluvial and deltaic deposits in this Value Addition: river. Among the mineral belts, North Eastern belts of Indian Peninsula are the richest

Prelim IAS Test Series (2019) – GS Test 12 (24.02.2019) 10 Geography 2 and Current Affairs Sep, Oct, Nov, Dec 2018 mineral belt of India followed by the The Deccan region is a large plateau in Central Belt. 90% of mineral deposits are western and southern India. It rises to found in Chota Nagpur belt. 100 metres in the north, and to more Source: Vajiram and Ravi Indian than 1,000 metres in the south, forming a Geography Chapter 11: Minerals and raised triangle within the Indian Industries, Page no. 223-231 subcontinent's coastline. It is between three mountain ranges and extends over 18. Match List-I with List-II and select eight Indian states. the correct answer using the code Malabar region refers to the geographic given below: area of southwest India. It spans from the List-I List-II south-western coast of Maharashtra and (Physical (Indian goes along the coastal region of Goa, Feature) Region) through the entire western coast of A. Himalayas 1 Deccan Karnataka and Kerala and reaches till . Kanyakumari. It is flanked by the Arabian B. Indo- 2 Rohilkhand Sea on the west and the Western Ghats Gangetic . on the east. Plains Source: NCERT Class 11th, India: C Peninsular 3 Malabar Physical Environment, Chapter 2: . Plateau . Structure and Physiography, Page 8- D West 4 Garhwal 16. . Coast . Plains 19. Which of the following factors Codes influence ‘South-West ’ in A B C D India? (a) 4 1 2 3 (1) Intense heating of Tibetan (b) 1 4 3 2 plateau (c) 1 2 3 4 (2) Westerly Jet Stream (d) 4 2 1 3 (3) Tropical Easterly Jet Stream Select the correct answer using the Answer: (d) code given below: Explanation: (a) 1 only Garhwal is the western region in the (b) 2 and 3 only northern Indian state of Uttarakhand (c) 1 and 3 only which is home to the Garhwali people. (d) 1, 2 and 3 Lying in the Himalayas, it is bounded on the north by , on the east by Answer: (d) Kumaon region, on the south by Uttar Explanation: Pradesh state, and on the northwest by The south-west monsoon sets in over the st Himachal Pradesh state. The people of Kerala coast by 1 June and moves Garhwal are known as Garhwali and swiftly to reach Mumbai and Kolkata speak the Garhwali language. between 10th and 13th June. By mid Rohilkhand is a region which lies in July, southwest monsoon engulfs the north-western region of Uttar Pradesh entire subcontinent. Various factors state, named after the Rohilla Afghan influence South-West Monsoon in India. tribes. Rohilkhand lies on the upper Factor 1 is correct. The Tibetan plateau alluvial plain. is intensely heated during summer and

Prelim IAS Test Series (2019) – GS Test 12 (24.02.2019) 11 Geography 2 and Current Affairs Sep, Oct, Nov, Dec 2018 thereby establishes an atmospheric Source: Vajiram and Ravi Geography circulation that is conducive for the Yellow book Indian Geography, monsoon. As the vast Tibetan plateau, Chapter 4: Climate, Page no. 64 high up in the mountains, gets warmed up during the summer months, it heats 20. Chambal basin is famous for which the air above. Such warm air rises and of the following features? hence creates an area of low pressure (1) Rectangular drainage pattern over Tibetan Plateau. This belt of low (2) Badland topography pressure sucks in moisture laden winds (3) Jog waterfall from the oceans, thus initiating the south- (4) Bird's foot delta west monsoon. Select the correct answer using the Factor 2 is correct. Sub-tropical code given below: Westerly Jet Stream is responsible for (a) 1 only the onset of the monsoon. Until the end (b) 2 only of May, this jet stream supplies the wind (c) 2 and 3 only to the Gangetic plain which is heated up (d) 1, 2 and 3 intensely as the sun is directly above the Tropic of Cancer (during the summer in Answer: (b) Northern hemisphere). These winds Explanation: supplied by this jet stream prevent the Chambal basin is famous for badland formation of a low pressure area in the topography. A region with a large number Gangetic plain. When sub-tropical of deep gullies or ravines is called westerly jet stream moves towards north badland topography. Such ravines are by the end of May, it no longer provides widespread, in the Chambal basin. In the winds to the Gangetic plain. This case of Chambal River and its many results in the formation of a Low tributaries, river rejuvenation occurred Pressure area in the Gangetic plain, not once or twice but multiple times. This which starts attracting the South East along with the semi arid climate and the Trade winds of the southern hemisphere. soil texture helped in creating a vast When such winds enter the northern network of ravines in the Chambal river hemisphere, they bend by 90 degrees basin in Madhya Pradesh, Rajasthan and due to the change in direction of the Uttar Pradesh. Chambal Badlands are Coriolis force in the northern hemisphere. extremely dissected, difficult to cross and This marks the onset of the South West agriculturally unfit. Monsoon. Source: NCERT XI – India: Physical Factor 3 is correct. The easterly jet Environment, Chapter 6: Soils, Page stream steers the tropical depressions no 73 into India. These depressions play a significant role in the distribution of 21. Consider the following statements monsoon rainfall over the Indian regarding ‘Deccan Traps’ of India: subcontinent. The tracks of these (1) They constitute flood basalt depressions are the areas of highest spread over large stretches rainfall in India. The frequency at which of land. these depressions visit India, their (2) Because of their igneous direction and intensity, all go a long way origin, they lack fossils. in determining the rainfall pattern during Which of the statements given the southwest monsoon period. above is/are correct?

Prelim IAS Test Series (2019) – GS Test 12 (24.02.2019) 12 Geography 2 and Current Affairs Sep, Oct, Nov, Dec 2018 (a) 1 only volcanic land forms and both animal and (b) 2 only plant life appeared in these areas. This is (c) Both 1 and 2 indicated by the occurrence of inter- (d) Neither 1 nor 2 trappean layers found in the lower and upper flows and their fossil contents. Answer: (a) Source: (1) Vajiram and Ravi - Yellow Explanation: Book, Indian Geography Chapter 2: The Deccan Traps are one of the largest Physiography of India, Page no. 26 volcanic provinces in the world, located (2)http://volcano.oregonstate.edu/decca on the Deccan Plateau of west-central n-traps India. The term "trap" refers to the step- like hills forming the landscape of the 22. In the last few years, India has region. Their formation began at the end made a quantum jump in ‘Solar of the Cretaceous period. Energy Capacity’. Consider the Statement 1 is correct: Deccan Traps following statements in this regard: constitute multiple layers of solidified (1) Among Indian states, flood basalt that together are more than Rajasthan has the largest 2,000 m thick, formed through series of solar energy potential eruptions. Deccan Traps cover much followed by Jammu & larger than 50,000 km2 and these lava Kashmir. piles formed the plateau topography. (2) At present, Tamil Nadu has Statement 2 is incorrect: The Deccan the highest installed solar Traps are famous for the beds of fossils energy capacity among the that have been found between layers of Indian states. lava. Many animal and plant fossils have Which of the statements given above is/are correct? (a) 1 only (b) 2 only (c) Both 1 and 2 (d) Neither 1 nor 2

Answer: (a) Explanation: India being a tropical country has 300 solar days and high potential for solar power.

been found from the inter-trappean beds of the Deccan Traps. The lava flows, especially after the beginning and before the close of activity, passed through definite periods of no volcanic activity. Statement 1 is correct. Among Indian During such periods; streams and lakes states, Rajasthan (142.31 GW) has the appeared on the cooled and congealed

Prelim IAS Test Series (2019) – GS Test 12 (24.02.2019) 13 Geography 2 and Current Affairs Sep, Oct, Nov, Dec 2018 largest solar energy potential followed by capillary action, which results in the Jammu & Kashmir (111.05 GW). deposition of salt on the top layer of the Statement 2 is incorrect. Karnataka is soil. In such areas, farmers are advised the top solar state in India. Karnataka to add gypsum to solve the problem of represents 24 percent market share of salinity in the soil. the total installed solar energy capacity in Source: NCERT XI – India: Physical India as of March 2018. Environment, Chapter 6: Soils, Page Value Addition: no 72 Domestic manufacturing of solar cells and modules in India is being supported 24. Arrange the following events in the by the Government of India through order of their occurrence in India: Modified Special Incentive Package (1) Establishment of first Jute Scheme (M-SIPS) of the Ministry of Mill Electronics & Information Technology. (2) Introduction of Railways The scheme, inter alia, provides for: (3) Setting up of Tata Iron and  20-25% subsidy for investments in Steel Plant in Jamshedpur capital expenditure for setting up of (4) Start of first Coal Mine the manufacturing facility. Select the correct answer using the  Reimbursement of counter-vailing code given below: Duty (CVD)/ Excise Duty for capital (a) 1-2-4-3 equipment for the units outside (b) 4-1-2-3 Special Economic Zone (SEZ). (c) 4-2-1-3 Source: Vajiram and Ravi Indian (d) 1-4-2-3 Geography Chapter 12: Energy Resources, Page no. 265 Answer: (c) Explanation: 23. Why are the farmers in agricultural Industrial development in India can be regions of Punjab and Haryana divided into two phases: advised to add gypsum to the soil? ● 1947-1980 – control of the (a) To kill the unwanted weeds Government from the crop lands. ● 1980-97 – Process of liberalisation (b) To accelerate the process of took over nitrogen fixation in soil. Some landmark events were: (c) To solve the problem of ● Coal mining started at Raniganj in salinity in the soil. 1772 (d) To increase the proportion of ● Railways were introduced in 1853 magnesium and sodium in ● First Cotton textile mill at Mumbai in the soil. 1854 ● Jute Industry made its beginning with Answer: (c) the first jute mill established in Explanation: 1855 in the Hooghly Valley In the areas of intensive cultivation with ● Tata Iron and Steel Plant was set up excessive use of irrigation, especially at Jamshedpur in 1907. green revolution areas like Punjab and Source: Vajiram and Ravi Yellow Haryana, the fertile alluvial soils are Book- Indian Geography, Chapter 11: becoming saline. Excessive irrigation Minerals and Industries, Page no. 237 with dry climatic conditions promotes

Prelim IAS Test Series (2019) – GS Test 12 (24.02.2019) 14 Geography 2 and Current Affairs Sep, Oct, Nov, Dec 2018 25. Irrigation in India includes a (1) Black soil network of canals, wells, tanks, etc. (2) Laterite soil Which of the following sources has (3) Arid soil maximum contribution in irrigated (4) Alluvial soil regions of India? Select the correct answer using the (a) Canals code given below: (b) Wells and Tubewells (a) 2-1-3-4 (c) Tanks (b) 4-3-1-2 (d) Multipurpose projects (c) 4-1-3-2 (d) 2-3-1-4 Answer: (b) Explanation: Answer: (c) Sources of irrigation in India: Explanation: Wells and Tubewells – 60%. It is more On the basis of genesis, colour, widespread in plains, coasts and some composition and location, the soils of regions of peninsular India. It is less India have been classified into: costly and more flexible as water can be drawn whenever needed and ‘evaporation loss’ is minimised and there is no fear of “over irrigation”. Uttar Pradesh leads in well irrigation followed by Punjab, Haryana, Bihar, Gujarat and Andhra Pradesh. Canals – 30%. Canals are one of the main sources of irrigation in India. Canals are big water channels taken out from rivers to carry water to places far away from the river. Punjab and Haryana have become the first granaries of country due to these canals. For example: Western Yamuna Canal, Sirhind Canal, etc. Tanks – 8%. It is prevalent in the uneven and relatively rocky plateau of Peninsular India. Tanks are commonly used in Deccan Plateau, Andhra Pradesh, Karnataka, Tamil Nadu and Maharashtra. Multipurpose projects – 4%. Example: i. Alluvial soils Beas Project ii. Black soils Source: Vajiram and Ravi Indian iii. Red and Yellow soils Geography Chapter 9: Agriculture, iv. Laterite soils Page no. 177 v. Arid soils vi. Saline soils 26. Arrange the following Indian soil vii. Peaty soils types in descending order of their viii. Forest soils coverage area (expressed as a Alluvial Soils: They are widespread in percentage of Indian geographical the northern plains and the river valleys. area): These soils cover about 40% of the total

Prelim IAS Test Series (2019) – GS Test 12 (24.02.2019) 15 Geography 2 and Current Affairs Sep, Oct, Nov, Dec 2018 area of the country. They are (a) 1 and 2 only depositional soils, transported and (b) 1, 3 and 4 only deposited by rivers and streams. (c) 2, 3 and 4 only Black Soil: They cover most of the (d) 1, 2, 3 and 4 Deccan Plateau which includes parts of Maharashtra, Madhya Pradesh, Gujarat, Answer: (b) Andhra Pradesh and some parts of Tamil Explanation: Nadu. These soils cover about 15% of Since Nagpur city is placed at the the total area of the country. These soils geographical centre of the country, the are also known as the ‘Regur Soil’ or the zero mile marker is located there. The ‘Black Cotton Soil’. Arid Soils: These soils are developed under arid and semi-arid conditions which occupy about 4.3% of the total area of the country. They are spread over Rajasthan, Saurashtra, Kachchh, Haryana and South Punjab. They range from red to brown in colour and are generally sandy in structure and saline in nature. Lower horizons of the soil are occupied by ‘kankar’ layers which restricts the infiltration of water. Laterite Soil: They cover around 3.7% of distances to various destinations across the total area of the country. These are the country are measured from this well developed on the summits of marker. The marker site was identified in Western Ghats, Eastern Ghats, the Great Trigonometrical Survey of Rajmahal hills, Satpuras and Vindhyas. India, undertaken by the East India They also occur in lower levels and in Company and the British Administration. valleys. The laterite soils develop in From the map given below, it is clear that areas with high temperature and high Bhopal, Hyderabad and Bengaluru cities rainfall characterised by alternate wet are located to the west of Nagpur city. and dry season which leads to leaching Raipur is located to the east of Nagpur away of siliceous matter of the rocks. city. Source: NCERT Class XI India Physical Environment– Chapter 6: 28. With reference to ‘Lakes’ in India, Soils, Page no. 69-70 consider the following statements: (1) Wular Lake is the largest 27. Which of the following cities are freshwater lake in India. located on latitudes, which are (2) Chilka Lake is the largest situated to the west of city Nagpur wintering ground for (State - Maharashtra)? migratory birds in India. (1) Bhopal (3) Sambhar Lake is the largest (2) Raipur inland saline lake of India. (3) Hyderabad Which of the statements given (4) Bengaluru above is/are correct? Select the correct answer using the (a) 2 only code given below: (b) 1 and 2 only

Prelim IAS Test Series (2019) – GS Test 12 (24.02.2019) 16 Geography 2 and Current Affairs Sep, Oct, Nov, Dec 2018 (c) 2 and 3 only Select the correct answer using the (d) 1, 2 and 3 code given below: (a) 1 and 2 only Answer: (d) (b) 2 only Explanation: (c) 3 only Wular Lake – Wular Lake is the largest (d) 1, 2 and 3 freshwater lake in India and lies in the Kashmir Valley, 40 km northwest of Answer: (d) Srinagar City in the Northwest of India. Explanation: With a size of 189 sq. km, Wular Lake is In Indian context, urbanisation is one of also one of the largest freshwater lakes the greatest phenomena of the latter half in Asia. of the 20th century. It began to Chilka Lake – Chilka Lake is located in accelerate after independence, due to eastern Odisha state of India. It is the country's adoption of a mixed separated from the Bay of Bengal by a economy. The process of society’s narrow spit. It is one of India’s largest transformation from a predominantly rural saltwater lakes. Chilka Lake is the largest to a predominantly urban population is wintering ground for migratory birds, known as ‘Urbanisation’. Top 10 especially flamingos, on the Indian Urbanized States of India area as shown subcontinent. Sambhar Lake – Located around 75 kilometres from Jaipur in Rajasthan, Sambhar lake is India’s largest inland saline lake and contributes significantly to the country’s salt production. The area turns home to an annual population of migratory birds from as far away as Europe and also local migratory species that thrive and nest in its waters teeming with algae and small fish. Among these, the greater and lesser flamingos are perhaps the most popular. Source: Orient Black Swan School Atlas, Page no: 10 http://www.walkthroughindia.com/attr in the table given. action/the-10-most-beautiful-lakes-in- Source: (1) Vajiram and Ravi - Yellow india/ Book, Indian Geography, Chapter 15: Urbanization in India, Page no. 338 29. As per Census 2011, which of the https://www.census2011.co.in/facts/to following Indian States/UTs are pstateurban.html categorized as most urbanized regions? 30. Which among the following states (1) Goa have their population density more (2) Delhi than the population density of (3) Kerala India? (1) West Bengal

Prelim IAS Test Series (2019) – GS Test 12 (24.02.2019) 17 Geography 2 and Current Affairs Sep, Oct, Nov, Dec 2018 (2) Kerala Select the correct answer using the (3) Arunachal Pradesh code given below: (4) Haryana (a) 1 and 2 only Select the correct answer using the (b) 2 only code given below: (c) 3 only (a) 1 and 4 only (d) None of the above (b) 2 and 3 only (c) 1, 2 and 4 only Answer: (c) (d) 1, 2 and 3 only Explanation: Gold is a precious metal but in India at Answer: (c) present the reserves of gold ore are low. Explanation: However, few regions where gold The population density of India has gone deposits can be found are as follows: up to 382 persons per square kilometre Karnataka – Kolar, Hutti, Topuldedi and in 2011 from 325 persons per square Wondalli. The Hutti gold mines are kilometre in 2001. owned by the government of Karnataka. States with very high density are located Kolar and Hutti fields together produce in Satluj-Ganga Plains, while more than 98% of country’s total mountainous Himalayan States have production of gold. Hence, pair 1 is incorrect but pair 3 is correct. Andhra Pradesh – The Ramagiri Gold field mine is situated in Anantpur district of Andhra Pradesh. Hence pair 2 is incorrect. Value Addition: Jharkhand – River beds of Garranadi, Sonanadi, Subarnrekha Source: Vajiram and Ravi Indian Geography Chapter 11: Minerals and least population density. Peninsular Industries, Page 228 States except Kerala (859) and Tamil Nadu (555) are marked with moderate 32. The Amarkantak Hill is the source density. of two rivers flowing in two different Source: Vajiram and Ravi - Yellow directions. These rivers are Book, Indian Geography, Chapter 14: (a) Narmada and Tapi Population, Page no: 308 (b) Narmada and Son (c) Tapi and Betwa 31. Gold deposits are found in India at (d) Tapi and Son only a few places. In this context, which of the following pairs are Answer: (b) correctly matched? Explanation: Field Location The Narmada, the largest west flowing 1. Kolar field : Tamil river of the Peninsula, rises near Nadu Amarkantak range of mountains in 2. Ramagiri : Bihar Madhya Pradesh. field The Son originates near Amarkantak in 3. Hutti field : Karnataka Madhya Pradesh, just east of the

Prelim IAS Test Series (2019) – GS Test 12 (24.02.2019) 18 Geography 2 and Current Affairs Sep, Oct, Nov, Dec 2018 headwaters of the , and (2) Garo, Khasi and Jaintia hills flows north-northwest through Madhya are structurally a part of the Pradesh state before turning sharply Himalayas. eastward. Which of the statements given above is/are correct? (a) 1 only (b) 2 only (c) Both 1 and 2 (d) Neither 1 nor 2

Answer: (a) Explanation: The Himalayas consist of a series of parallel mountain ranges. Himalayas are Source: Vajiram and Ravi - Yellow not only the physical barrier; they are Book, Indian Geography Chapter 3: also a climatic, drainage and cultural Drainage System and River Water, divide. Page no. 39-44 Statement 1 is correct: The Greater Himalayas (Himadri) is the northern-most 33. India’s Western Ghats and Eastern and highest of all the Himalayan ranges. Ghats meet at It is the only range of the Himalayas (a) Mahadeo Hills which maintains its continuity from west (b) Garhjat Hills to east. The approximate length of the (c) Nallamalla Hills Great Himalayan range, also known as (d) Nilgiri Hills the central axial range, is 2,500 km from east to west, and their width varies Answer: (d) between 160-400 km from north to south. Explanation: Statement 2 is incorrect: Garo, Khasi The Nilgiri Hills, far south in Tamil Nadu and Jaintia in Meghalaya are structurally is the meeting point of the Western a part of peninsular plateau and not the Ghats and the Eastern Ghats. It contains Himalayan ranges. It is believed that due several lofty peaks, the highest of which, to the force exerted by the north- , is 2634 meters above the eastward movement of the Indian plate at sea level. the time of the Himalayan origin, a huge Source: Vajiram and Ravi - Yellow fault was created between the Rajmahal Book, Indian Geography Chapter 2: hills and the Meghalaya plateau. Later, Physiography of India, Page no. 23-25 this depression got filled up by the deposition activity of the numerous 34. Consider the following statements rivers. Today, the Meghalaya and Karbi regarding the ‘Himalayan Mountain Anglong plateau stand detached from the System’ of India: main Peninsular Block (1) The Greater Himalayas is the Source: Vajiram and Ravi Yellow only range of the Himalayas Book, Indian Geography, Chapter 2: which maintains its continuity Physiography of India; Page no. 14- from west to east. 15-16-17

Prelim IAS Test Series (2019) – GS Test 12 (24.02.2019) 19 Geography 2 and Current Affairs Sep, Oct, Nov, Dec 2018 35. Match the names of major Indian Cochin port is one of the largest port in ports with their locations (as shown India and the major port on the Arabian in the Sea and Indian Ocean sea route. map) Source: Vajiram and Ravi - Yellow Book, Indian Geography, Chapter 13: Transport and Communication Network, Page no: 296

36. Consider the following statements with reference to ‘Shale Gas’: (1) Shale gas is a colourless and odourless gas. (2) Shale gas releases less CO2 than coal during combustion. Names: (3) Andhra Pradesh has A. Kandla maximum number of shale B. Ennore gas blocks for exploration. C. Paradip Which of the statements given D. Cochin above is/are correct? Code: (a) 1 and 2 only A B C D (b) 3 only (a) 1 2 4 3 (c) 2 and 3 only (b) 1 4 2 3 (d) 1, 2 and 3 (c) 4 3 2 1 (d) 3 2 4 1 Answer: (a)

Answer: (b) Explanation: Kandla Port is situated on the Gulf of Kutch in Kutch District of Gujarat. The Port of Kandla is the first special economic zone in India as well as in Asia. Explanation: Ennore Port, officially renamed For meeting the country’s growing Kamarajar Port Limited, is located on the energy needs, exploration and Coromandel Coast about 24 km north of exploitation of shale gas is very Chennai Port, Chennai. It is the 12th essential. major port of India. The Kamarajar Port is Statement 1 is correct. Shale gas is the only corporatized major port and is colourless, odourless gas. It is lighter registered as a company. than air. Paradip Port is an artificial, deep-water Statement 2 is correct. Shale gas port of eastern coast of India which is releases 50% less CO2 than coal hence it located in the Odisha state. It is the is an environment friendly source for major port situated at the confluence of generating electricity. great river and the Bay of Statement 3 is incorrect. Maximum Bengal. Shale gas blocks are in Gujarat (28) followed by Andhra Pradesh (10).

Prelim IAS Test Series (2019) – GS Test 12 (24.02.2019) 20 Geography 2 and Current Affairs Sep, Oct, Nov, Dec 2018 Cambay, KG basin, Cauvery, Ganga and (2) Large scale deforestation is Assam-Arakan regions are also rich in the primary challenge faced Shale gas in India. by the Majuli Island. Source: Vajiram and Ravi Yellow Which of the statements given Book, Indian Geography, Chapter 12: above is/are not correct? Energy Resources, Page no. 258 (a) 1 only (b) 2 only 37. Consider the following statements (c) Both 1 and 2 with reference to a particular (d) Neither 1 nor 2 ‘National Park’ of India: (1) It is a biodiversity hotspot in Answer: (b) the Eastern Himalayas. Explanation: (2) It is famous for a particular Assam's famed Majuli island, which species of flying squirrel. recently won the title of being the world's (3) It is located in Arunachal largest river island, is declared India's Pradesh. first island district. The 880 square km The description given above is Majuli island on the Brahmaputra River most appropriate for which one of displaced Marajo island of Brazil to the following Indian National become the world's largest river island Parks? according to Guinness World Records. (a) Kaziranga National Park Statement 1 is correct: (b) Namdapha National Park Geomorphologically, the entire Majuli (c) Keibul Lamjao National Park Island is a part of the alluvial flood plains (d) Dudhwa National Park of the Brahmaputra River. The Island is formed of soil consisting mainly of silt Answer: (b) deposits. Explanation: Statement 2 is incorrect: Soil Erosion Namdapha National Park, located in and Floods are the pressing challenges Arunachal Pradesh in north-east India is faced by the Majuli Island. Majuli had an the largest protected area in the Eastern area of 1,256 sq km in 1991 but due to Himalaya biodiversity hotspot. It is also soil erosion, it has now reduced to 875 the third largest national park in India in sq km. The soil of Island is without terms of area. Namdapha flying squirrel cohesion and thus, susceptible to is an arboreal, nocturnal flying squirrel erosion. Also, Brahmaputra is one of the endemic to north-eastern India where it is most difficult rivers to tackle as it is very known from a single specimen collected dynamic due to frequent geomorphology in Namdapha National Park in 1981. changes. Flood control and anti-soil Source: Orient Black Swan School erosion measures adopted elsewhere Atlas, Page no. 35 may not be applicable in this river which is 3rd largest in the world and carries 38. Consider the following statements highest silt load amongst the rivers of its about ‘Majuli Island’: size. (1) Geomorphologically, the Source: NCERT XI, India – Physical entire Majuli Island is a part Environment, Chapter 2: Structure of the alluvial flood plains of and Physiography, Page no 13 – 14. the Brahmaputra River.

Prelim IAS Test Series (2019) – GS Test 12 (24.02.2019) 21 Geography 2 and Current Affairs Sep, Oct, Nov, Dec 2018 39. Consider the following Indian (a) South-Central Railway rivers: (b) North-Central Railway (1) Indus (c) North-Eastern Railway (2) Sutlej (d) East-Central Railway (3) Brahmaputra (4) Indravati Answer: (c) Which of the above rivers Explanation: represent antecedent drainage For administrative convenience and system? efficient transportation, railways were (a) 1, 2 and 3 only (b) 2, 3 and 4 only (c) 1, 2 and 4 only (d) 1, 3 and 4 only

Answer: (d) Explanation: Antecedent streams are those which are originated prior to the upliftment of land surface. In other words, antecedent streams antedate the upliftment of an upland or mountain across which they have maintained their present courses through continuous downcutting of their valleys. The nature and the rate of upliftment of land area is very important parameter for the development of antecedent drainage system. Examples grouped into railway zones. Gorakhpur is of antecedent streams are found in the headquarters of North-Eastern almost all of the Fold Mountains of the Railway Zone. world. Many of the major Himalayan Source: Vajiram and Ravi - Yellow rivers are the examples of antecedent Book, Indian Geography, Chapter 13: streams e.g., the Indus, the Sutlej, the Transport and Communication Ganga, the Ghaghra, the Kali, the Network, Page no: 287 Gandak, the Kosi, the Brahmaputra etc. : It is a tributary of the river 41. The plain region at the foothills of Godavari. Its starting point is found to be the ‘Shiwalik Mountains’ where the Ghats of Dandakaranya range, in the heavy boulders, gravels and Kalahandi district of the state of Odisha. coarse sediments flown in by It does not represent antecedent Himalayan rivers are deposited is drainage system. called as Source: NCERT XI India: Physical (a) Tarai Environment, Chapter 3: Drainage (b) Doon System, Page no 25, 26 (c) Khadar (d) Bhabar 40. Gorakhpur is the headquarters of which of the following Railway Answer: (d) Zones of India? Explanation:

Prelim IAS Test Series (2019) – GS Test 12 (24.02.2019) 22 Geography 2 and Current Affairs Sep, Oct, Nov, Dec 2018 The northern plains of India are formed fields by ONGC in 1970s, Natural Gas by the alluvial deposits brought by the assumed importance. Later discoveries Himalayan rivers – the Indus, the Ganga were made in Gujarat, Krishna-Godavari and the Brahmaputra. From the north to basin, Cauvery basin, Tripura, Assam the south, the plains can be divided into etc. three major zones: the Bhabar, the Tarai Source: Vajiram and Ravi - Yellow and the Alluvial plains. The flat plains Book, Indian Geography, Chapter 12: along the sub-Himalayan region in North Energy Resource, Page no. 258 India are called Bhabar. It is a narrow belt ranging between 8-10 km parallel to 43. Which of the following species the Shiwalik foothills. As a result of this, is/are found in Sundarbans forest the streams and rivers coming from the located in West Bengal state of mountains deposit heavy materials of India? rocks and boulders, and at times, (1) Royal Bengal Tiger disappear in this zone. South of bhabar (2) Crocodile is the tarai belt with an approximate width (3) Gangetic Dolphin of 15-30 km where most of the streams Select the correct answer using the and rivers re-emerge without having any code given below: properly demarcated channel, thereby (a) 1 only creating marshy and swampy conditions. (b) 1 and 2 only Source: Vajiram and Ravi Yellow (c) 2 and 3 only Book, Chapter 2: Physiography of (d) 1, 2 and 3 India; Page no. 19 Answer: (d) 42. Which of the following places in Explanation: India have known reserves of The Sundarbans is a cluster of low-lying natural gas? islands in the Bay of Bengal, spread (1) Assam across India and Bangladesh, famous for (2) Krishna-Godavari Basin its unique mangrove forests. This active (3) Cauvery Basin delta region is among the largest in the Select the correct answer using the world, measuring about 40,000 sq km. code given below: The Sundarban forest is about 10,000 sq (a) 1 and 2 only km across India and Bangladesh, of (b) 2 and 3 only which 40% lies in India, and is home to (c) 1 and 3 only many rare and globally threatened (d) 1, 2 and 3 wildlife species such as the estuarine crocodile, royal Bengal tiger (Panthera Answer: (d) tigris), Water monitor lizard, Gangetic Explanation: dolphin, and Olive Ridley turtle. Gas is one of the cleanest fuels with less Source: carbon dioxide per joule delivered than NCERT VII Chapter 8: Tropical and either by coal or oil and far fewer Subtropical Regions, Page no. 61 pollutants than other hydrocarbon fuels. https://www.wwfindia.org/about_wwf/critic The development of Natural Gas industry al_regions/sundarbans3/about_sundarba in the country started in 1960s with ns/ discovery of gas fields in Assam and Gujarat. After discovery of South Basin

Prelim IAS Test Series (2019) – GS Test 12 (24.02.2019) 23 Geography 2 and Current Affairs Sep, Oct, Nov, Dec 2018 44. Which of the following pairs are 45. Consider the following statements correctly matched? with reference to ‘Tectonic Activity’ River Tributary in India: 1. Indus : Zaskar (1) The Indian plate is moving at 2. Yamuna : Son a speed of 1 km per year 3. Godavari : Manjra towards the north and north- 4. Krishna : Amravati eastern direction. Select the correct answer using the (2) The Indian peninsula is code given below: tectonically stable as it has (a) 1 and 3 only not witnessed any seismic (b) 1, 2 and 3 only activity in the past. (c) 2 and 4 only Which of the statements given (d) 1, 3 and 4 only above is/are not correct? (a) 1 only Answer: (a) (b) 2 only Explanation: (c) Both 1 and 2 A river basin is a part of land through (d) Neither 1 nor 2 which a main river and all its tributaries flows. It is the basic hydrological unit for Answer: (c) planning and development of water Explanation: resources of a country. Statement 1 is incorrect. The Indian Pair 1 is correctly matched: Some plate is moving at a speed of one Himalayan tributaries of Indus River are centimetre per year towards the north Shyok, Gilgit, Zaskar, Hunza, Nubra, and north-eastern direction and this Shigar and Dras. The Chenab is the movement of plates is being constantly largest tributary of the Indus. obstructed by the Eurasian plate from the Pair 2 is incorrectly matched: The north. As a result of this, both the plates tributaries of Yamuna River are Chambal, are said to be locked with each other Sind, Betwa and Ken on its right bank, resulting in accumulation of energy at while Hindan, Rind, Sengar, Varuna, etc. different points of time. Excessive join Yamuna on its left bank. Note: Son accumulation of energy results in building is a south bank tributary of the Ganga, up of stress, which ultimately leads to the originating in the Amarkantak plateau. breaking up of the lock and the sudden Pair 3 is correctly matched: The release of energy, causes an earthquake. principal tributaries of River Godavari are Statement 2 is incorrect. High stresses Penganga, Indravati, Pranhita, and generated by the continental collision, Manjra. caused by the northward movement of Pair 4 is incorrectly matched: The India, have affected the entire Indian major tributaries of River Krishna are Peninsula. Due to intense pressure, the Koyna, Tungbhadra and Bhima. peninsula has up-warped in many Note: Amravati is a tributary of River places. And worse, ancient rifts that had . criss-crossed the nation's sub-surface Source: NCERT class XI, India strata, are now suddenly being Physical Environment, Chapter 3: reactivated. Many geologists now believe Drainage System, Page no- 24-28 that the Latur 1993 earthquake was caused by the reactivation of just such a rift zone. The earthquake in Latur has

Prelim IAS Test Series (2019) – GS Test 12 (24.02.2019) 24 Geography 2 and Current Affairs Sep, Oct, Nov, Dec 2018 shaken conventional theories. What it 47. Which of the following pairs is/are proves is that we can expect a 6.5 correctly matched? magnitude earthquake anywhere in the Channel Feature peninsula. It means that the so called 1 10 : Separates the Indian shield that extends from . degree Andaman Kanyakumari to the Himalayan foothills is Channel Islands and vulnerable to major seismic activity. Nicobar Source: NCERT – XI India: Physical Islands Environment, Chapter 7: Natural 2 9 degree : Separates Hazards and Disasters, Page no. 80-81 . Channel Minicoy Island from the rest of 46. A low density, yet uncontrolled, the expansion of larger urban areas Lakshadweep into surrounding agricultural areas 3 8 degree : Separates is called as . Channel Great Nicobar (a) Out Growth island and (b) Urban Agglomeration Little Nicobar (c) Satellite Town Island (d) Urban Sprawl Select the correct answer using the code given below: Answer: (d) (a) 1 only Explanation: (b) 1 and 2 only An urban agglomeration is a continuous (c) 1 and 3 only urban spread constituting a town and its (d) 2 and 3 only adjoining outgrowths (OGs), or two or more physically contiguous towns Answer: (b) together with or without outgrowths of Explanation: such towns. Pair 1 is correct: The Ten Degree An Out Growth (OG) is a viable unit such Channel separates the Andaman Islands as a village or a hamlet or an and Nicobar Islands from each other in enumeration block made up of such the Bay of Bengal. village or hamlet and clearly identifiable Pair 2 is correct: Minicoy Island is in terms of its boundaries and location. separated from Lakshadweep by 9 Satellite Towns are suburban centres degree channel. providing cheaper and spacious Pair 3 is incorrect: Lakshadweep is alternative to main city. separated from Maldives by 8 degree An Urban Sprawl is a low density channel. uncontrolled expansion of larger urban Source: Vajiram and Ravi Yellow areas into surrounding agricultural areas. Book, Chapter 1: Indian: General It happens because of rapid growth of Introduction; Page no. 6 city’s population and widening range of economic activities. 48. Consider the following statements Source: Vajiram and Ravi - Yellow in the context of India’s Book, Indian Geography, Chapter 15: geographical extent: Urbanization in India, Page no: 340 (1) The mainland of India extends from Arunachal

Prelim IAS Test Series (2019) – GS Test 12 (24.02.2019) 25 Geography 2 and Current Affairs Sep, Oct, Nov, Dec 2018 Pradesh in the east to Mizoram signifies the land of the Lushai Rajasthan in the west. highlanders. It is located in the southern (2) The north-south extent of corner of Northeast India nestled mainly India is more than its east- between Myanmar and Bangladesh, but west extent. also shares a part of its boundaries with Which of the statements given its neighbouring states of Assam, above is/are correct? Manipur and Tripura. (a) 1 only NCERT XI – India: Physical (b) 2 only Environment, Chapter 2: Structure (c) Both 1 and 2 and Physiography, Page No. 15 (d) Neither 1 nor 2 50. With reference to fertilizers in India, Answer: (b) consider the following statements: Explanation: (1) Phosphatic and Potassic Statement 1 is incorrect. The mainland fertilisers have been of India extends from Kashmir in the decontrolled since 1992. north to Kanniyakumari in the south and (2) At a national level, Nitrogen Arunachal Pradesh in the east to Gujarat (N): Phosphorous (P): in the west. India’s territorial limit further Potassium (K) ratio of 4:2:1 is extends towards the sea upto 12 nautical considered as an optimum miles (about 21.9 km) from the coast. fertilizer consumption ratio. Statement 2 is correct. In context of Which of the statements given India’s extent, the actual distance above is/are correct? measured from north to south extremity (a) 1 only is 3,214 km, and that from east to west is (b) 2 only only 2,933 km. (c) Both 1 and 2 Source: NCERT XI – India: Physical (d) Neither 1 nor 2 Environment, Chapter 1: India – Location, Page No. 2 Answer: (c) Explanation: 49. Which of the following states is Most of the fertilisers used in India have also known as the ‘Molassis to be imported either in raw material or Basin’? complete form. The burden of (a) Mizoram continuously rising prices of fertilisers (b) Manipur falls to a great extent on the Government (c) Nagaland of India. (d) Meghalaya Statement 1 is correct. In keeping with the policy of liberalisation and reforms, Answer: (a) Phosphatic and Potassic fertilisers have Explanation: been decontrolled since 1992. Mizoram is a landlocked state in North Statement 2 is correct. At a national East India. Mizoram is also known as the level, the Nitrogen (N): Phosphorous (P): ‘Molassis basin’ which is made up of soft Potassium (K) ratio of 4:2:1 is considered unconsolidated deposits. The state of as an optimum ratio. Mizoram is popularly known as the Value Addition: songbird of India and is one of the But the current consumption ratio of smallest states in the country. The name nitrogen, phosphorus and potassium

Prelim IAS Test Series (2019) – GS Test 12 (24.02.2019) 26 Geography 2 and Current Affairs Sep, Oct, Nov, Dec 2018 (NPK) is 6.7:2.4:1. The situation is launched on 31 March, 1958. The canal grimmer in major agricultural states like begins at the Harike Barrage - at the Punjab and Haryana where NPK use confluence of the Sutlej and Beas rivers ratios are as high as 31.4:8.0:1 and 27.7: in the Indian Punjab - and continues in a 6.1:1 respectively. The deterioration in south-westerly direction for some 290 fertilizer mix will not only have an impact miles (470 km). on productivity of crops but also on long- Pair 2 is incorrect. Buckingham canal term soil health. (258 km) along coastal Andhra Pradesh Source: Vajiram and Ravi Yellow is part of National Waterway (NW)-4. The Book, Indian Geography, Chapter 9: Buckingham Canal is a manmade, Agriculture, Page no. 175. saltwater, navigation canal that runs parallel to the Coromandel Coast in the 51. Canal irrigation is well developed in north-south direction. certain regions of India. In this Pair 3 is correct. Mutha Canal has been context, which of the following pairs taken out from the river Mutha near Pune are correctly matched? in Maharashtra. The canal starts from Canal Area Khadakwasla, flows across the city and Served further travels towards Phursungi. 1 Indira : Rajasthan Value Addition: Gandhi ● Sampad Sagar Canal – Godavari Canal ● Sirhind canal – Satluj 2 Buckingham : West ● Eden canal – Damodar Canal Bengal ● Anicut Canal - Kaveri 3 Mutha : Maharashtr Source: Vajiram and Ravi Yellow Canal a Book, Indian Geography, Chapter 9: Select the correct answer using the Agriculture, Page 179 code given below: (a) 1 only 52. Which of the following Indian rivers (b) 1 and 3 only flows through a rift valley? (c) 2 and 3 only (a) Bhima (d) 1, 2 and 3 (b) Kaveri (c) Indravati Answer: (b) (d) Damodar Explanation: S. Answer: (d) Canal Name Area served No. Explanation: The drains the eastern Indira Gandhi Punjab, 1 margins of the Chota nagpur plateau. It Canal Rajasthan flows through a rift valley and finally joins Buckingham Andhra Pradesh, the Hugli. The Damodar valley, along the 2 Canal Tamil Nadu Bihar–West Bengal border includes India’s most important coal and mica 3 Mutha Canal Maharashtra mining fields and has long been an area Pair 1 is correct. Indira Gandhi Canal, of active industrial development. The previously known as the Rajasthan river is famous all over the world for the Canal, is one of the largest canal Damodar Valley Corporation (DVC). The systems in India. The canal project was length of the river is 592 km. The river

Prelim IAS Test Series (2019) – GS Test 12 (24.02.2019) 27 Geography 2 and Current Affairs Sep, Oct, Nov, Dec 2018 traverses important cities like Asansol, south, parallel to the Coromandel Coast on the Bay of Bengal. Javadi hills are located on the Eastern Ghats in North Arkat district of Tamil Nadu.

Bokaro, Durgapur, etc. Note: Other rivers which flow through a rift valley are Narmada and Tapti. Source: Vajiram and Ravi - Yellow Book, Indian Geography, Chapter 3: Drainage System and River Water, Page no. 40

53. Which of the following represents the correct sequence of hills when you travel (from north to south) along ‘Eastern Ghats’ in India? (a) Javadi Hills – Anaimalai Hills – Cardamom Hills – Nallamala Hills (b) Nallamala Hills – Javadi Hills Anaimalai Hills are extended in the – Anaimalai Hills – north-south direction on the borders of Cardamom Hills Kerala and Tamil Nadu to the south of (c) Javadi Hills – Nallamala Hills the Nilgiri Hills. – Cardamom Hills – Cardamom Hills are located to the south Anaimalai Hills of the Annamalai hills. (d) Nallamala Hills – Javadi Hills Orient BlackSwan School Atlas – India – Cardamom Hills - Anaimalai Physical, Page No.10 Hills 54. Which of the following pairs is/are Answer: (b) correctly matched? Explanation: National Located in Nallamala Hills are located south of the Parks of Indian State . The hills run north to India 1. Kishtawar : Jammu and

Prelim IAS Test Series (2019) – GS Test 12 (24.02.2019) 28 Geography 2 and Current Affairs Sep, Oct, Nov, Dec 2018 Kashmir mining in the area of Sariska National 2. Dudhwa : Assam park in 1991. Sariska is the first tiger 3. Sariska : Rajasthan reserve in the world to have successfully Select the correct answer using the relocated tigers. code given below: Few National Parks of North India: (a) 1 only Orient BlackSwan School Atlas – India (b) 3 only Physical, Page No. 35 (c) 1 and 2 only (d) 1 and 3 only 55. With reference to ‘Drought in India’, Trikal is considered as most Answer: (d) devastating due to its adverse Explanation: effects. Which of the following can Pair 1 is correctly matched: Jammu be included under such adverse and Kashmir - Salim Ali, Dachigam, effects? Kishtawar, Hemis National Park. (1) Scarcity of food grains, fodder and water. (2) An epidemic due to spread of cholera, gastro-enteritis and heptatis. (3) Outward migration of productive population from villages. (4) Increase in soil erosion by wind, water and grazing animals. Select the correct answer using the Kishtwar National Park: This national code given below: park was basically made to protect the (a) 1 and 3 only endangered species of snow leopards (b) 1, 2 and 3 only and was given the status of a national (c) 1, 3 and 4 only park in 1981. It is located in the Kishtwar (d) 1, 2, 3 and 4 district of the state of Jammu and Kashmir. Answer: (b) Pair 2 is incorrectly matched: Uttar Explanation: Pradesh – Dudhwa National Park, Droughts have cascading effects on Chandraprabha Wildlife Sanctuary. various other aspects of environment and Dudhwa National park is located in the society. state of Uttar Pradesh. It was established Point 1 is correct: Crop failure leading as National Park in the year of 1977. This to scarcity of food grains (akal), fodder region comes under the Terai belt. (trinkal), inadequate rainfall, resulting in Pair 3 is correctly matched: Rajasthan shortage of water (jalkal), and often - Desert, Keoladeo Ghana, Sariska, shortage in all the three (trikal) is most Ranathambor National Park. devastating. Sariska National Park: It is located in Point 2 is correct: Scarcity of water state of Rajasthan. The area of Sariska is compels people to consume rich in mineral resources, such as contaminated water resulting in spread of copper. Supreme Court of India banned

Prelim IAS Test Series (2019) – GS Test 12 (24.02.2019) 29 Geography 2 and Current Affairs Sep, Oct, Nov, Dec 2018 many waterborne diseases like gastro- Point 3 is correct. The lion-tailed enteritis, cholera, hepatitis, etc. macaque is an Old World monkey Point 3 is correct: Large-scale death of endemic to the Western Ghats of South cattle and other animals, migration of India. humans and livestock are the most Point 4 is incorrect. In India, Red common sight to be seen in the drought Panda is found in Sikkim, western affected areas. Arunachal Pradesh, Darjeeling district of Point 4 is incorrect: Due to inadequate West Bengal and parts of Meghalaya. It rainfall, soil erosion is not possible by is also the state animal of Sikkim. Listed water. as Endangered in the IUCN red list of Source: NCERT XI – India: Physical Threatened Species and under Schedule Environment, Chapter 7: Natural I of the Indian Wildlife (Protection) Act, Hazards and Disasters, Page 88-89-90 1972, the red panda has the highest legal protection at par with other 56. Which among the following can be threatened species. found in the ‘Nilgiri Biosphere Source: Vajiram and Ravi - Yellow Reserve’ of India? Book, Indian Geography, Chapter 6: (1) Nilgiri Tahr Natural Vegetation and Wildlife. (2) Indian Bustard (3) Lion - Tailed Macaque 57. Which of the following pairs is/are (4) Red Panda correctly matched? Select the correct answer using the Indian Origin code given below: River (a) 1 and 3 only 1. Kosi : Tribeni Ghat (b) 2 and 3 only 2. Kaveri : Brahamagiri (c) 3 and 4 only Hills (d) 2, 3 and 4 only 3. Chenab : Lahaul Spiti Select the correct answer using the Answer: (a) code given below: Explanation: (a) 1 only Point 1 is correct. The Nilgiri tahr (b) 1 and 2 only inhabits the open montane grassland (c) 2 and 3 only habitats at elevations from 1200 to 2600 (d) 1, 2 and 3 m (generally above 2000 m) of the South Western Ghats. Their significant Answer: (d) concentration is in the Nilgiri Hills, with Explanation: smaller populations in the Anamalai Hills, Pair 1 is correct: originates , Palni Hills. from Tribeni Ghat in Nepal. The river Point 2 is incorrect. Historically, the Kosi consists of seven streams and is great Indian bustard was distributed popularly called as Saptkoshi in Nepal. throughout Western India, spanning 11 The river is braided and shifts its course states, as well as parts of Pakistan. Its frequently. This has resulted in frequent stronghold was once the Thar desert in devastating floods. Thus, the river is the north-west and the Deccan plateau of often termed as the “Sorrow of Bihar”. the peninsula. Today, its population is Pair 2 is correct: Kaveri River originates confined mostly to Rajasthan and from Tala Kaveri in Brahamgiri hills Gujarat. located in Coorg plateau of Karnataka.

Prelim IAS Test Series (2019) – GS Test 12 (24.02.2019) 30 Geography 2 and Current Affairs Sep, Oct, Nov, Dec 2018 The river descends from South Karnataka plateau to Tamil Nadu plains through famous Sivasamudram waterfall. The Kaveri has formed quadrilateral delta in Bay of Bengal. Pair 3 is correct: Chenab originates near Bara Lacha Pass in the Lahul Spiti part of Zaskar Range. The Chandra and Bhaga are the two main tributaries of river Chenab. Source: Vajiram and Ravi Yellow Book, Chapter 3: Drainage System and River Water; Page no. 39 and 44 feed from the nutrients on the bed, the phumdis are thinning out and even 58. Lake Loktak is famous for floating breaking away. Worse, local residents biomass islands i.e Phumdis. themselves are breaking off pieces of the Phumdis, a rich source of biomass to sell elsewhere as rich fish vegetation is thinning out and even culture soil. breaking away. The main reason Value Addition: behind this issue is In the heart of the Loktak Lake is the (a) Siltation has decreased the largest Phumdi, the 40 sq. km Keibul water level in the lake. Lamjao National Park, home to the (b) Hydroelectric project endangered brown-antlered deer, the constantly keeps high water sangai, whose habitat is under threat just level in the lake. as the islands are. (c) Large scale deforestation has Source: Orient Black Swan School caused an increase in Atlas, Page no. 35 surface run-off. (d) Unsustainable grazing by 59. Consider the following map: Hangul.

Answer: (b) Explanation: Lake Loktak (Manipur) stretches to about 500 sq. km during the rainy season and reduces to nearly half in the dry months of February and March. It is famous for the floating biomass islands or Phumdis. The Phumdis float during the rainy season. They sink during the dry months, sucking nutrients from the lake bed to replenish their roots and float again when the next monsoon cycle begins. This has Which of the following Indian rivers been their life cycle for centuries but now is represented by the stretch A-B they are at grave risk as the Loktak as shown in the map? Hydroelectric Project constantly keeps (a) Luni the water level in the lake high. Unable to (b) Mahi

Prelim IAS Test Series (2019) – GS Test 12 (24.02.2019) 31 Geography 2 and Current Affairs Sep, Oct, Nov, Dec 2018 (c) Betwa 61. Match List I with List II and select (d) Tapi the correct answer by using the codes given below: Answer: (a) List I List II Explanation: (Tree (State/UT) Luni originates from the Naga Hills of the Species) Aravalli Range in Rajasthan. Luni does A. Teak 1. Andaman & have freshwater during the first hundred Nicobar kilometres but as it reaches Balotra in Islands Barmer, the water starts to turn saline B. Deodar 2. Rajasthan from the rich salt content of the surface it C Mahogany 3. Odisha flows on. Luni then flows into the Thar . Desert and meets its end in Barine, D Khair 4. Himachal located at the north-eastern part of the . Pradesh marsh called the Rann of Kutch in Codes: Gujarat, without flowing into any larger A B C D water body. (a) 1 3 2 4 Source: Orient Black Swan School (b) 3 4 1 2 Atlas, Page no. 10 (c) 2 4 1 3 (d) 4 3 1 2 60. The colour of which of the following soils is linked with the wide Answer: (b) diffusion of iron in ancient Explanation: crystalline and metamorphic rocks? Forest Type (a) Black Soil Tropical Evergreen Forests (b) Laterite Soil Trees: Rosewood, Mahogony, Aini, (c) Red soil Ebony, etc. (d) Alluvial Soil Location: Western slope of the Western ghats, hills of the north-eastern region Answer: (c) and the Andaman and Nicobar Islands. Explanation: Tropical Moist Deciduous Forests Red soil is an important soil resource, Trees: Teak, sal, shisham, hurra, mahua, which bears substantial implication for amla, semul, kusum, sandalwood, etc. sustainable development of agriculture Location: North-eastern states along the and healthy growth of economy. Red soil foothills of Himalayas, eastern slopes of contains a high percentage of iron the Western Ghats and Odisha. content, which is responsible for its Tropical Thorn Forests colour. It is formed by the weathering of Trees: Khair, neem, khejri, palas, ancient crystalline and metamorphic Tussocky grass, etc. rocks, particularly acid granites and Location: Semi-arid areas of south west gneisses, quartzitic rocks, and felspathic Punjab, Haryana, Rajasthan, Gujarat, rocks. Madhya Pradesh and Uttar Pradesh. Source: Vajiram and Ravi - Yellow Montane Forests Book, Indian Geography, Chapter 5: Trees: Oak, chestnut, chir pine, Deodar, Land and Soil Resources, Page no. 90 chinar, walnut, blue pine, spruce, silver firs, junipers, pines, birch and rhododendrons, etc.

Prelim IAS Test Series (2019) – GS Test 12 (24.02.2019) 32 Geography 2 and Current Affairs Sep, Oct, Nov, Dec 2018 Location: Himalayan ranges which Zaskar range: It forms northern wall of include states of Jammu and Kashmir, the Zaskar and Suru valleys and the Himachal Pradesh, Uttarakhand, North- southern wall of the Indus valley. eastern States. Pir Panjal Range: It is a group of Southern mountain forests include the mountains in the Inner Himalayan region, Western Ghats, the Vindhyas and the running across the Indian states of Nilgiris. Himachal Pradesh and Jammu and Source: NCERT India Physical Kashmir. Environment Class XI – Chapter 5, page no. 57-61

62. Arrange the following mountain ranges of ‘Kashmir Himalayas’ in order of their occurrence (from north to south) on the map of India. (1) Pir Panjal (2) (3) Ladakh (4) Zaskar Select the correct answer using the code given below: (a) 4 – 3 – 1 – 2 (b) 1 – 4 – 3 – 2 (c) 1 – 3 – 4 – 2 (d) 2 – 3 – 4 – 1

Answer: (d) Explanation: Ranges from north to south are as Source: Vajiram and Ravi Yellow follows: Karakoram – Ladakh – Zaskar Book, Chapter 1: Indian- General – Pir Panjal. Introduction; Page no. 13 Kashmir or North-Western Himalayas: It comprises a series of ranges such as 63. With reference to ‘Mangroves’, the Karakoram, Ladakh, Zaskar and Pir consider the following statements: Panjal. The Kashmir Himalayas are also (1) Most extensive area of famous for Karewa formations, which are mangroves is found in South useful for the cultivation of Zafran, a local East Asia. variety of saffron. (2) India accounts for nearly Karakoram Range: K2, the second 3.3% of the world’s highest peak of the world, at 8,611m lies mangrove vegetation. in the Karakoram Range. Hindu-Kush, an (3) Sundarbans accounts for half extension of the Karakoram Range runs of the total area under in Afghanistan. Mangroves in India. Ladakh range: It forms northern wall of Which of the statements given the Indus valley and the southern wall of above is/are correct? the Nubra and Shyok valleys. It extends (a) 3 only down into Tibet. (b) 1 and 2 only

Prelim IAS Test Series (2019) – GS Test 12 (24.02.2019) 33 Geography 2 and Current Affairs Sep, Oct, Nov, Dec 2018 (c) 1 and 3 only Popularly known as the Dakshin Ganga, (d) 1, 2 and 3 the Godavari is the largest peninsular river system. The Godavari originates Answer: (d) from Nasik district of Maharashtra and Explanation: discharges its water into the Bay of Mangroves are plants that survive high Bengal. With total 1,465 km length, salinity, tidal regimes, strong wind Godavari covers the areas of velocity and muddy anaerobic soil – a Maharashtra, Madhya Pradesh, combination of conditions hostile for Chhattisgarh, Odisha, etc. The other plants. Not all coastal areas are Penganga, the Indravati, the Pranhita, suitable for mangrove plantation as and the Manjra are the major tributaries mangroves require an appropriate mix of of Godavari. saline and freshwater and soft substrate like mudflats to be able to grow and perpetuate. Statement 1 is correct: Most extensive area of mangroves is found in South East Asia (33%) followed by South America (15%). The total Mangrove cover in the world is 150,000 sq.km approximately. Statement 2 is correct: Mangroves are Source: NCERT XI India: Physical spread over an area of 4921 sq. km in Environment, Chapter 3: Drainage India which is nearly 3.3 % of the world’s System, Page no. 27 mangrove vegetation. Statement 3 is correct: Sundarban 65. Madhya Pradesh has the largest accounts for half of the total area under forest cover in the country in terms Mangrove in India. The Sundarban of absolute area. In terms of mangrove forest lies on the delta of the percentage of forest cover with Ganges, Brahmaputra and Meghna respect to the total geographical rivers on the Bay of Bengal. area, which of the following Indian Source: (1) Vajiram and Ravi - Yellow states/ U.Ts has the highest forest Book, Indian Geography, Chapter 6: cover? Natural Vegetation and Wildlife, Page (a) Arunachal Pradesh no. 99 (b) Mizoram (2) http://fsi.nic.in/isfr2017/isfr- (c) Andaman and Nicobar Island mangrove-cover-2017.pdf (d) Lakshadweep

64. Which of the following Indian rivers Answer: (d) is also known as Dakshin Ganga? Explanation: (a) Kaveri Madhya Pradesh (77,414 sq. km) has the (b) Godavari largest forest cover in the country in (c) Tungbhadra terms of area, followed by Arunachal (d) Krishna Pradesh (66,964 sq. km) and Chhattisgarh (55,547 sq. km). Answer: (b) In terms of percentage of forest cover Explanation: with respect to the total geographical area, Lakshadweep with (90.33%) has

Prelim IAS Test Series (2019) – GS Test 12 (24.02.2019) 34 Geography 2 and Current Affairs Sep, Oct, Nov, Dec 2018 the highest forest cover, followed by By early June, the inflow of the south- Mizoram (86.27%) and Andaman and west monsoon into India brings about a Nicobar Island (81.73%). change in the weather. The maximum Source: (1) Vajiram and Ravi - Yellow rainfall of this season is received in the Book, Indian Geography, Chapter 6: north-eastern part of the country. Natural Vegetation and Wildlife, Page Mawsynram in the southern ranges of no. 103 the Khasi Hills receives the highest (2) average rainfall in the world. http://pibphoto.nic.in/documents/rlink/201 Early in the season, the windward side of 8/feb/p201821201.pdf the Western Ghats receives very heavy rainfall i.e more than 250cm. The Deccan 66. Which of the following species has Plateau and parts of Madhya Pradesh witnessed a catastrophic also receive some amount of rainfall in population decline in the past due spite of lying in the rain shadow area. to use of livestock drug Tamilnadu coast receives rainfall during ‘Diclofenac’? the season of retreating monsoon. (a) Indian Vulture Source: NCERT Class IX – (b) Indian Bustard Contemporary India, Chapter 4: (c) Indian Pitta Climate, Page no. 33 (d) Indian Swiftlet 68. Which among the following cash Answer: (a) crops are grown in India? Explanation: (1) Tobacco Widespread use of diclofenac in Indian (2) Wheat cattle is linked to the deaths of millions of (3) Cotton vultures that ate carcasses containing (4) Rice the drug, causing their populations to Select the correct answer using the decline by more than 99% since the code given below: 1990s. Diclofenac is toxic to vultures (a) 1 and 2 only even in small doses, causing kidney (b) 1 and 3 only failure. (c) 1, 2 and 4 only Source: (d) 1, 3 and 4 only https://www.ncbi.nlm.nih.gov/pubmed/ 29159701 Answer: (b) Explanation: 67. The maximum rainfall during the Agricultural crops which are grown with ‘South-West Monsoon’ season is an objective of making profit are called received by which of the following as Cash Crops. Such crops have a good regions of India? demand and a ready market for sale. In (a) Western Ghats India, cash crops like sugarcane, (b) Deccan Plateau tobacco, tea, rubber, coffee, cotton etc (c) Tamilnadu Coast are grown. (d) Southern Ranges of the Note: Wheat is a Rabi crop and Rice is Khasi Hills Kharif crop. Source: NCERT Class VII, Chapter 8: Answer: (d) Tropical and Subtropical regions, Explanation: Page no: 61

Prelim IAS Test Series (2019) – GS Test 12 (24.02.2019) 35 Geography 2 and Current Affairs Sep, Oct, Nov, Dec 2018 3. Dihang Dibang: Biosphere reserve 69. Identify the biosphere reserves located in Arunachal Pradesh exhibits which are marked (as 1, 2 and 3) natural vegetation stretching in an on the India Map given below. unbroken sequence from the tropics to the mountain tundra. Value Addition: Dibru Saikhowa: This Biosphere Reserve is located in the flood plains of Brahmaputra plains of Assam. It is located in one of the 19 biodiversity hotspots of the world. Nilgiri: It was the first biosphere reserve in India established in the year 1986.The reserve is situated in the Western Ghats, in the Nilgiri Hills of South India and is considered as an International Biosphere Reserve. The Nilgiri Biosphere Reserve was declared as World Heritage site in 2012. Pachmarhi: This Biosphere Reserve is located in Central India. The Satpura Select the correct answer using the mountain ranges cross India from west to code given below: east and Pachmarhi lies directly at its (a) Sesachalam Hills, centre. Achanakmar-Amarkantak, Source: Orient Black Swan School Dibru Saikhowa Atlas, Page no. 35 (b) Nilgiri, Simlipal, Dibru Saikhowa 70. With respect to Census 2011 of (c) Sesachalam Hills, India, consider the following Achanakmar-Amarkantak, statements: Dihang Dibang (1) Decadal sex ratio of total (d) Nilgiri, Pachmarhi, Dihang population has continuously Dibang decreased since 1961. (2) Decadal sex ratio of children Answer: (c) aged 0-6 years has Explanation: continuously increased since 1. Seshachalam: The hill ranges 1961. spread in the parts of Chittoor and Which of the statements given Kadapa districts of Andhra Pradesh above is/are not correct? have been designated as (a) 1 only Seshachalam Biosphere Reserve. (b) 2 only 2. Achanakmar-Amarkantak: This (c) Both 1 and 2 biosphere reserve is located in (d) Neither 1 nor 2 Chhattisgarh and Madhya Pradesh State. It is home to tigers, panthers, Answer: (c) bears, Chitals, sambars, barking Explanation: deers and bison (gaur).

Prelim IAS Test Series (2019) – GS Test 12 (24.02.2019) 36 Geography 2 and Current Affairs Sep, Oct, Nov, Dec 2018 Such rivers flow with lesser velocity before entering into sea. Factor 2 is incorrect. Short course of rivers doesn’t provide much scope to collect large amount of slit. Generally, east flowing rivers like Godavari, Krishna which form deltas have a long course. Factor 3 is correct. East flowing rivers have enough tributaries and big Statement 1 is incorrect: Decadal Sex catchment area. Hence, they are laden ratio of total population increased in with huge amount of sediments eroded 1981, 1991 and 2011. by such tributaries over their long course. Statement 2 is incorrect: Decadal Sex Factor 4 is incorrect. Very few east ratio of children aged 0-6 years has flowing rivers traverse through rift valley. continuously decreased since 1961. Rift Valley swiftly drains whatever sediments are carried by such rivers into 71. Generally, deltas are seen on the the sea without forming any delta. eastern coast and not on the Source: Vajiram and Ravi - Yellow western coast of India. Which of Book, Indian Geography, Chapter 3: the following factors are mainly Drainage system and River Water, responsible for such contrasting Page no. 42 feature of the Indian coasts? (1) East flowing rivers flow over 72. Identify the type of Indian forest gentle slope of Indian indicated by “X” marks in the peninsula. (2) East flowing rivers have a short course. (3) East flowing rivers carry huge loads of sediments. (4) East flowing rivers flows through a rift valley. Select the correct answer using the code given below: (a) 1 and 2 only (b) 1 and 3 only (c) 2 and 4 only (d) 1, 2 and 4 only

Answer: (b) Explanation: Factor 1 is correct. Slight tilting of the following map: peninsular block from northwest to the south-eastern direction gave an (a) Moist deciduous forest orientation to the entire drainage system (b) Montane forest towards the Bay of Bengal. Gentle slope (c) Tropical evergreen forest means more deposits thus bigger deltas. (d) Littoral forest

Prelim IAS Test Series (2019) – GS Test 12 (24.02.2019) 37 Geography 2 and Current Affairs Sep, Oct, Nov, Dec 2018 Answer: (c) 74. Which of the following can act as Explanation: serious development challenges to Tropical Evergreen Forest: These the Indian economy? forests are found in the hills of the north- (1) Inequality between different eastern region, western slope of the states Western Ghats and the Andaman and (2) Inequality within the states Nicobar Islands. They are found in warm themselves and humid areas with an annual (3) Inequality between the rural precipitation of over 200 cm and mean and urban areas annual temperature above 22֯C. The (4) Inequality within the rural semi-evergreen forests are also found in societies and the urban the lesser rainy parts of these regions. societies themselves Such forests have a mixture of evergreen Select the correct answer using the and moist deciduous trees. code given below: Source: NCERT India Physical (a) 1 and 2 only Environment Class XI – Chapter 5, (b) 1 and 3 only page no. 57-58 (c) 1, 2 and 3 only (d) 1, 2, 3 and 4 73. Which of the following soil types is mainly developed in the areas of Answer: (d) Peninsular Plateau experiencing Explanation: high rainfall and high temperature? With its wide diversities in physiography, (a) Red Soil history, demography and sociology, India (b) Peaty Soil has been characterised by regional (c) Alluvial Soil disparities in socioeconomic (d) Laterite Soil development not only between States but also between districts of a State and Answer: (d) between areas and social groups within Explanation: districts. Following are the basic types of The laterite soils develop in areas with regional disparities in India: high temperature and high rainfall. These  Inter-state inequality (Developed are the result of intense leaching due to Maharashtra state vs tropical rains. These soils have mainly Underdeveloped North-Eastern developed in the higher areas of the states) peninsular plateau. The laterite soils are  Inequality within the states commonly found in Karnataka, Kerala, themselves (Underdeveloped Tamil Nadu, Madhya Pradesh and the Vidarbha region within developed hilly areas of Odisha and Assam. Maharashtra state) Laterites are not suitable for cultivation;  Inequality between the rural and the however, application of manures and urban areas (Underdeveloped fertilisers are required for making the Melghat vs developed Mumbai) soils fertile for cultivation.  Inequality within the rural societies Source: NCERT XI – India: Physical and the urban societies themselves Environment, Chapter 6: Soils, Page Such regional disparities present a no 71 serious development challenge to the Indian economy. They hinder the progress towards ‘Faster, More Inclusive

Prelim IAS Test Series (2019) – GS Test 12 (24.02.2019) 38 Geography 2 and Current Affairs Sep, Oct, Nov, Dec 2018 and Sustainable Growth’. Regional sea surface temperatures are greater disparities in India can act as a counter than normal in the Arabian Sea and less current to the ideas of ‘National than normal in the tropical eastern Indian Integration and Federalism’ thus Ocean. A positive IOD leads to greater adversely affect the socio-economic monsoon rainfall and more active (above indicators of the whole country. normal rainfall) monsoon days while Source: Vajiram and Ravi – Yellow negative IOD leads to less rainfall and Book, Indian Geography – Chapter 16: more monsoon break days (no rainfall). Regional Development and Planning, Source: (1) NCERT XI – India: Physical Page no. 357 Environment, Chapter 4: Climate, Page no. 38 (2) 75. Consider the following statements: https://www.thehindu.com/sci- (1) Generally, El Nino and the tech/science/The-dipole-factor-in- Indian Monsoon are inversely summer-monsoon- related. rainfall/article14479847.ece (2) A negative Indian Ocean Dipole leads to less rainfall 76. Recently, Munroe Thuruthu Island and more monsoon breaks in was in news. It is reported to be India. sinking due to rising sea levels Which of the statements given attributed to climate change. This above is/are correct? island is located in (a) 1 only (a) Andaman and Nicobar (b) 2 only Islands (c) Both 1 and 2 (b) Tamil Nadu (d) Neither 1 nor 2 (c) Kerala (d) Lakshadweep Answer: (c) Explanation: Answer: (c) Statement 1 is correct: Generally, El Explanation: Nino and the Indian Monsoon are inversely related. Conventional El Niño is characterized by strong anomalous warming in the eastern equatorial Pacific. During El-Nino event, warming of the Pacific Ocean results in the weakening of Trade winds. Therefore, moisture and heat content gets limited and results in reduction and uneven distribution of rainfall across the Indian sub-continent. Statement 2 is correct: The Indian summer monsoon rainfall is influenced by a system of oscillating sea surface temperatures known as the Indian Ocean Dipole (IOD) in which the western Indian Ocean becomes alternately warmer and Munroe Thuruthu in district, then colder than the eastern part of the Kerala is slowly going under water, and ocean. A positive IOD occurs when the its fleeing residents join the swelling

Prelim IAS Test Series (2019) – GS Test 12 (24.02.2019) 39 Geography 2 and Current Affairs Sep, Oct, Nov, Dec 2018 ranks of climate refugees. Munroe (a) They signal river floods by Thuruthu is a string of eight islets at the making sound. confluence of the Ashtamudi Lake and (b) They remove the sediments the Kallada River. The island can be from the river. seen as the first casualty of global (c) They eat away the algal warming in Kerala, creating a band of bloom of a river. environmental refugees. The island’s (d) They act as bio-indicators of decline began with the construction of aquatic pollution. the Thenmala dam. Located 70 km away and constructed in the 1960s under the Answer: (d) Kallada Irrigation Project, the dam Explanation: blocked the flow of fresh water as a Top predators, such as mammalian result of sediments from the Kallada river carnivores, sea birds, and raptors, are — the main determinant of the land’s among the widely used indicator species. fertility. Today the whole area has turned Top predators tend to be concentrated in saline. important biodiversity hotspots. The reduction or disappearance of top 77. Consider the following pairs of predators is related to significant Volcanoes (which were in news ecosystem transformations, including recently) and their respective impacts on several trophic levels and location: changes in energy flows, over- Volcano Location exploitation of resources, and changes in 1. Mt. Krakatoa : Indonesia the behaviour of prey or food chain 2. Mt. Etna : Cyprus structure. Moreover, their presence or 3. Mt. Soputan : Italy absence can indicate the extent of the Which of the pairs given above footprint of human pressures. River is/are correctly matched? dolphins, also known as 'susu' are top (a) 1 only predators and can act as bio-indicators of (b) 1 and 2 only aquatic pollution of freshwater on the (c) 2 and 3 only basis of three ecological indicators: (i) (d) 1, 2 and 3 density of river dolphins, (ii) mean group size of dolphins, and (iii) dolphin sighting Answer: (a) rates. Explanation: 2018 witnessed some of the deadliest 79. Which of the following is volcanic eruptions. Following volcanoes responsible for nearly 65% of CO2 were in news recently: emissions in India? ● Mt. Krakatoa: Indonesia (a) Natural Gas ● Mt. Etna: Italy (b) Cement ● Mt. Soputan: Indonesia (c) Oil Hence, only pair 1 is correctly (d) Coal matched. Answer: (d) 78. ‘Susu’, normally found in the Explanation: freshwaters of the River Ganga, Although, India is rapidly going in for are of great significance because solar and wind power, coal usage continues to grow strongly. Coal is

Prelim IAS Test Series (2019) – GS Test 12 (24.02.2019) 40 Geography 2 and Current Affairs Sep, Oct, Nov, Dec 2018 responsible for 65% of India's CO2 revealed that the OPV, made by a emissions. Ghaziabad-based firm called Bio-Med, Limiting global warming to the 2015 Paris contained traces of the Type 2 vaccine Agreement goal of keeping the global virus. temperature increase this century, to well Reason 2 is correct. OPV itself can below 2°C, would need carbon dioxide cause polio in rare cases. There are two emissions to decline by 50% by 2030 ways in which all three oral vaccine and reach net zero by about 2050. viruses can cause polio. The first is called Vaccine Associated Paralytic Polio 80. All oral polio vaccines (OPV) (VAPP). The second way in which the across the world contain only two vaccine can cause polio is through of the three polio serotypes - Type Circulating Vaccine Derived Polio Virus 1 and Type 3 where as Type 2 is (cVDPV). banned. Which of the following statements is/are the primary 81. The ‘Food Sustainability Index’ is reasons behind such a ban? based on the following three broad (1) The wild Type 2 virus was parameters: eradicated globally by 1999. (1) Food loss and waste (2) Oral Polio Virus itself can (2) Sustainable agriculture cause polio in rare cases. (3) Nutritional challenges Select the correct answer using the As per comparison with the global code given below: scenario, under which parameter(s) (a) 1 only mentioned above, India has (b) 2 only performed well? (c) Both 1 and 2 (a) 1 only (d) Neither 1 nor 2 (b) 2 only (c) 2 and 3 only Answer: (c) (d) 1, 2 and 3 Explanation: Polio vaccines are vaccines used to Answer: (a) prevent poliomyelitis (polio). Two types Explanation: are used: an inactivated poliovirus given A 'food system' involves the infrastructure by injection (IPV) and a weakened and processes that go into feeding the poliovirus given by mouth (OPV). All oral population, such as growing, harvesting polio vaccines (OPV) across the world and transportation. The Food contain only two of the three polio Sustainability Index (FSI) measures how serotypes — Type 1 and Type 3 where sustainable these processes are. as Type 2 is banned. Despite the increase in food production, Reason 1 is correct. Type 2 is banned the FSI ranks India 33rd among 67 because the wild, disease-causing countries in 2018. China does better than version of this virus was eradicated India, while other BRICS partners are globally by 1999. However, sometime in rated poorly. A comparison with the September 2018, routine surveillance global scenario indicates following things: detected the Type 2 vaccine virus in stool  India does well on preventing food samples from children in Uttar Pradesh, loss and wastage. Hence, Point 1 is implying that someone was still making correct the vaccine. Further investigations

Prelim IAS Test Series (2019) – GS Test 12 (24.02.2019) 41 Geography 2 and Current Affairs Sep, Oct, Nov, Dec 2018  It is below average in sustainable Statement 2 is incorrect. Nearly 80% of agriculture. Hence, Point 2 is the world's total proven natural gas incorrect reserves are located in ten countries.  India is one of the worst countries in Russia tops the list, holding about a terms of tackling nutritional quarter of world's total gas reserves, challenges. Hence, Point 3 is followed by Iran and Qatar in the Middle incorrect East.

82. Consider the following statements: 83. Recently, the term ‘Sharp Eye’ was (1) Among OPEC nations, in news which highlighted India’s though Venezuela has the progress in the field of indigenous most ‘Proven Reserves of oil’; technology. It denotes Saudi Arabia produces the (a) An infra-red imaging drone most crude oil per day. developed by DRDO for (2) In the year 2018, Qatar's surveillance of sensitive area ‘Proven Reserves of natural near Line of Control gas’ have become more than (b) The facial expression that of Russia. recognition algorithm Select the correct answer using the developed by IIT Chennai for code given below: artificial intelligence related (a) 1 only application. (b) 2 only (c) The most powerful camera (c) Both 1 and 2 ever developed by the ISRO (d) Neither 1 nor 2 to be used in space satellite. (d) None of the above Answer: (a) Explanation: Answer: (c) Statement 1 is correct. Saudi Arabia Explanation: has the second-largest reserves of India's Hyper Spectral Imaging Satellite naturally occurring oil in the world after (HysIS) is dubbed as ‘Sharp Eye’. HysIS Venezuela. The Kingdom of Saudi Arabia is the best-ever high-resolution earth produces a whopping 13.24 percent of all imaging satellite launched into space by the oil produced daily in the entire world. India. HysIS is a very rare satellite with a Saudi Arabia and Russia top the list of oil super-sharp eye, and very few countries exporting countries but Russia is not a have indigenously mastered this technology. "Sharp eye" denotes the most powerful camera ever developed by the ISRO for use in space. HysIS will enable better identification of objects on the surface of the subcontinent, and aid in forestry, agriculture and smart city mapping. Only the United States, China and the European Space Agency have systems as sophisticated as our hyper-spectral member of (Organization of the imaging camera. HysIS satellite will Petroleum Exporting Countries) OPEC. improve both day and night viewing

Prelim IAS Test Series (2019) – GS Test 12 (24.02.2019) 42 Geography 2 and Current Affairs Sep, Oct, Nov, Dec 2018 capabilities, giving India an edge over biggest emitters in 2018 are China, U.S., other countries. India, Russia, Japan, Germany, Iran, Saudi Arabia, South Korea, and Canada. 84. Adequacy of Foreign Exchange The EU as a region of countries ranks Reserve of a country is measured third. China’s emissions accounted for by 27% of the global total, having grown an (a) Debt to GDP Ratio estimated 4.7% in 2018 and reaching a (b) Import Cover new all-time high. (c) Export Cover (d) Debt to Foreign Exchange 86. GSAT-11 is India’s first six-tonne- Reserve Ratio class advanced satellite. It was put into orbit from the European Answer: (b) spaceport in Guiana in South Explanation: America. Its primary mission is to Import cover of reserves is the traditional (a) boost communication and trade-based indicator of foreign improve disaster links among exchange reserve adequacy. It tells us India’s six neighbouring how long imports can be sustained in the countries. event of a shock. According to the IMF, (b) provide hyper-spectral traditionally, the import coverage imaging services for a range measure has been based on months of of applications like prospective imports, with three months’ agriculture, forestry, etc. coverage typically used as a benchmark. (c) monitor weather and climate As per Reserve Bank of India, the of the Earth, especially global reserves, which stood at $424.54 billion warming effects. as at end-March 2018, fell to $405.74 (d) enable high-speed satellite- billion as at end-June 2018. Latest data based internet services to showed the reserves were at $392.7 users in rural, remote areas. billion by end of November 16, 2018 Answer: (d) 85. According to the findings of 'Global Explanation: Carbon Project', which among the India’s first six-tonne-class ‘big bird’ in following are the four biggest CO2 space, advanced communication satellite emitters? GSAT-11, was put into orbit from the (a) China, U.S., India, Japan European spaceport in Guiana in South (b) China, U.S., India, Germany America. The heaviest ever to be built by (c) China, U.S., European Union, the Indian Space Research Organisation India (ISRO), the 5,854-kg satellite was (d) China, U.S., India, Russia launched from the Guiana Space Centre at Kourou. Its mission is to enable high- Answer: (d) speed satellite-based Internet services to Explanation: users in rural, remote areas and to Global carbon emissions are set to hit an businesses down home over the next 15 all-time high of 37.1 billion tonnes of CO2 years. It will meet most of the in 2018 - 2019, according to researchers requirements of providing broadband at the University of East Anglia (UEA) connectivity to rural and inaccessible and the Global Carbon Project. The 10

Prelim IAS Test Series (2019) – GS Test 12 (24.02.2019) 43 Geography 2 and Current Affairs Sep, Oct, Nov, Dec 2018 village panchayats under Bharat Net (d) Light which is part of the Digital India initiative. Answer: (d) 87. Recently, the Reserve Bank of Explanation: India has relaxed the securitization Astronomical spectroscopy is the study norms by relaxing the minimum of astronomy using the techniques of holding period requirement. This spectroscopy to measure the spectrum of step was mainly targeted to provide electromagnetic radiation, including liquidity support to visible light, which radiates from stars (a) Foreign banks and other celestial objects. A (b) Real estate spectrograph is an instrument used to (c) Aviation Sector obtain and record an astronomical (d) Non-Banking Financial spectrum. The spectrograph disperses Institutions the light from an object into its component wavelengths so that it can be Answer: (d) recorded then analysed. Explanation: Recently, a team of Indian scientists and In a move to make more liquidity engineers discovered a sub-Saturn or available to non-banking finance firms, super-Neptune size planet around a Sun- the Reserve Bank of India has relaxed like star. The discovery was made using the securitization norms by relaxing the Spectrograph, PARAS to measure and minimum holding period requirement. confirm the mass of the new planet. The move follows a demand from the government for a special window for 89. To encourage inter-tribal interaction Non-banking financial Companies and to promote cultural heritage, a (NBFCs), to provide them liquidity 'Hornbill Festival' is organized support. every year in the first week of The NBFC sector is facing liquidity December. In which state of India, shortage after Infrastructure Leasing & this unique festival is organized? Finance Services, a core investment (a) Nagaland company, started defaulting on loans (b) Assam which resulted in the government (c) Tripura dismantling the existing board of IL&FS (d) Manipur and installing a new one. The cost of funds has gone up for the non-banking Answer: (a) finance firms putting pressure on Explanation: profitability. To encourage inter-tribal interaction and to promote cultural heritage of Nagaland, 88. Planets, stars and galaxies are just the Government of Nagaland organizes too far away to be analysed in a the Hornbill Festival every year in the first laboratory. Important aspects of week of December. It is also called the such distant bodies can most ‘Festival of Festivals”. The aim of the effectively be studied with the help festival is to revive and protect the rich of culture of Nagaland and display its (a) Solar Flares extravaganza and traditions. The Festival (b) Vacuum is named after the hornbill, the globally (c) Sound

Prelim IAS Test Series (2019) – GS Test 12 (24.02.2019) 44 Geography 2 and Current Affairs Sep, Oct, Nov, Dec 2018 respected bird and which is displayed in investor who wishes to invest folklore in most of the state’s tribes. in a new start up. (d) minimum time taken by 90. The Government of India has receivers on the Earth to started implementation of Dam detect signals coming from Rehabilitation and Improvement the dark side of the Moon. Project (DRIP) to rehabilitate 198 dam projects in 7 States with Answer: (b) financial assistance from Explanation: (a) Asian Infrastructure 'Environmental flows' or e-flows are the Investment Bank “minimum flow requirements” and as long (b) Asian Development Bank as flow is kept above a critical level, the (c) World Bank river ecosystem will be conserved. (d) New Development Bank Environmental flows are the acceptable flow regimes that are required to Answer: (c) maintain a river in the desired Explanation: environmental state or predetermined The Government of India has started state. implementation of Dam Rehabilitation The Central Government on October and Improvement Project (DRIP) with 10th 2018 notified the minimum financial assistance from World Bank to environmental flows for River Ganga that rehabilitate 198 dam projects in 7 States has to be maintained at various locations viz. Kerala, Madhya Pradesh, Odisha, on the river. The notification issued by Karnataka, Uttarakhand, Jharkhand and the Government will ensure that the river Tamil Nadu. has at least the minimum required Central Water Commission (CWC) environmental flow of water even after maintains National Register of Large the river flow gets diverted by projects Dams (NRLD) as per information and structures for purposes like irrigation, provided by State Govts/ Dam owners. hydropower, domestic and industrial use Prime responsibility of preparation of etc. This was an important step taken Disaster Management Plan (DMP)/ towards maintaining the uninterrupted Emergency Action Plan (EAP) rests with flow of the river. dam owners which are State Governments, central and state PSUs, 92. With reference to Eklavya Model municipalities and private companies etc. Residential Schools (EMRs), consider the following statements: 91. The term 'E-flow' was in news (1) For the promotion of recently. It means education among scheduled (a) minimum business per year caste (SC) students, EMRSs by an e-commerce company are set up in Indian States / to avoid bankruptcy. Union Territories (UTs). (b) minimum flow requirements (2) The establishment of EMRSs to maintain the river in a is based on demand of the desired environmental state. concerned Indian States/UTs (c) minimum investment with availability of land as an requirement for an angel essential attribute.

Prelim IAS Test Series (2019) – GS Test 12 (24.02.2019) 45 Geography 2 and Current Affairs Sep, Oct, Nov, Dec 2018 Which of the statements given Explanation: above is/are not correct? Central Silk Board (CSB) has notified (a) 1 only newly developed races of silkworm seed (b) 2 only of mulberry and Vanya silk for increasing (c) Both 1 and 2 the productivity of cocoons and to (d) Neither 1 nor 2 increase the income of the farmers engaged in sericulture. Silk worm breeds Answer: (a) for specific agro-climatic condition are Explanation: essential for increasing the productivity of Statement 1 is incorrect: In the context cocoons. of establishing quality residential schools Tropical Tasar Silkworm (BDR10) race for the promotion of education, Eklavya developed by CSB have 21% more Model Residential Schools (EMRSs) for productivity than the traditional Daba Scheduled Tribes (ST) students are set breed. Farmers can get upto 52 kg up in States / UTs with provisioning of cocoons per 100 disease free layings funds through “Grants under Article (dfls). This silkworm breed will help the 275(1) of the Indian Constitution”. tribal farmers of Jharkhand, Statement 2 is correct: The Chhattisgarh, Odisha, West Bengal, establishment of EMRSs are based on Andhra Pradesh, Maharashtra, Madhya demand of the concerned States/UTs Pradesh, Bihar, Telangana and Uttar with availability of land as an essential Pradesh. attribute. As per the budget 2018-19, Central Silk Board under the Ministry of every block with more than 50% ST Textiles is engaged in applied research population and at least 20,000 tribal for developing new breeds of races of persons, will have an Eklavya Model silkworm seed and conducts extensive Residential School by the year 2022. field trial before commercial use in the field. 93. 'Tropical Tasar (BDR10)' has a great significance for Indian 94. Recently, an autonomous institute Economy. It is a under Ministry of Science and (a) Minor Forest Produce (MFP) Technology has developed for which Minimum Support oligosaccharides-based coating. Price (MSP) has been This coating is primarily used to declared by Indian (a) Improve the stealth feature of Government recently. fighter jets. (b) Race of a silkworm which is (b) Improve the reflective newly developed by the potential of concrete building Central Silk Board of India. walls. (c) Species of butterfly which is (c) Enhance the absorption endemic to Western Ghats in capacity of solar panels. India. (d) Enhance the shelf-life of (d) Variety of drought resistant fruits and vegetables. rice which is newly developed by the National Rice Answer: (d) Research Institute, India. Explanation: National Agri-Food Biotechnology Answer: (b) Institute (NABI) is under the

Prelim IAS Test Series (2019) – GS Test 12 (24.02.2019) 46 Geography 2 and Current Affairs Sep, Oct, Nov, Dec 2018 administrative control of Department of Statement 2 is correct: National Biotechnology, Ministry of Science and Waterway 2 (NW-2) is a section of the Technology, Government of India. NABI Brahmaputra River having a length of has developed oligosaccharides-based 891 km between the Bangladesh border natural fruit coating that enhances the near Dhubri and Sadiya in Assam. It was shelf-life of fruits and vegetables. The declared as National Waterway No. 2 on technology has dual advantages, first it 1 September 1988. The NW 2 is one of utilizes the agriculture waste and second the major freight transportation waterway it can replace the shellac, a coating of north east India and the third longest material that is obtained from an insect. Waterway. Value Addition: NABI has demonstrated proof of concept 96. The Union Cabinet has approved in Banana for the feasibility of carrying “The DNA Technology (Use and out CRISPR based genome editing. This Application) Regulation Bill, 2018”. technology is being exploited to develop The Bill has been formulated for traits in Banana as well as in crops like the regulation of the use and wheat, rice and Lathyrus. application of Deoxyribonucleic Acid (DNA) technology. Which of 95. Consider the following statements: the following statements is correct (1) Inland Water Transport (IWT) about DNA? mode is an environment (a) DNA is a single stranded friendly and cost effective molecule. mode of transport. (b) DNA is not stable under (2) National Waterway-2 is a alkaline conditions. section of the Brahmaputra (c) DNA is self-replicating. River located in the Indian (d) DNA is not susceptible to UV state of Assam only. damage. Which of the statements given above is/are correct? Answer: (c) (a) 1 only Explanation: (b) 2 only Statement a is incorrect: DNA is a (c) Both 1 and 2 double-stranded molecule while RNA is a (d) Neither 1 nor 2 single stranded molecule. Statement b is incorrect: DNA is stable Answer: (c) under alkaline conditions while RNA is Explanation: not stable. Statement 1 is correct: Inland Water Statement c is correct: DNA is self- Transport (IWT) mode is widely replicating. RNA is synthesized from recognized as environment friendly and DNA on an as-needed basis. cost effective mode of transport. As per Statement d is incorrect: DNA is RITES Report of 2014 on Integrated susceptible to UV damage. Compared National Waterways Transportation Grid, with DNA, RNA is relatively resistant to some of the important benefits of IWT UV damage. mode compared to rail and road Value Addition: transportation are fuel efficiency and cost DNA and RNA perform different functions savings. in humans. DNA is responsible for storing and transferring genetic information while

Prelim IAS Test Series (2019) – GS Test 12 (24.02.2019) 47 Geography 2 and Current Affairs Sep, Oct, Nov, Dec 2018 RNA directly codes for amino acids and acts as a messenger between DNA and 98. ‘NAQUIM’ programme was in news ribosomes to make proteins. recently. The main objective of this Indian programme is to 97. Consider the following statements: (a) Enhance the capacity of (1) National Steel Policy 2017 States in ground water has set ambitious target of management and 300 MT steel capacity by development. 2030-31. (b) Enhance the quality (2) Steel Authority of India management framework at Limited (SAIL) is the largest Bureau of Indian Standards. steel producer and the (c) Monitor the quality of air in highest iron ore miner in Indian urban metro cities. India. (d) Monitor the improvement in Which of the statements given the quality of education at above is/are correct? elementary level. (a) 1 only (b) 2 only Answer: (a) (c) Both 1 and 2 Explanation: (d) Neither 1 nor 2 The National Project on Aquifer Management (NAQUIM) is an initiative of Answer: (c) the Ministry of Water Resources, River Explanation: Development & Ganga Rejuvenation, Statement 1 is correct: National Steel Government of India, for mapping and Policy 2017 has set ambitious target of managing the entire aquifer systems in 300 MT steel capacity by 2030-31. the country. The vision of the National Currently, the per capita steel Project on Aquifer Management is to consumption in India is 60kg. The policy identify and map aquifers at the micro seeks to raise it to 160Kg by 2030. SAIL level, to quantify the available with 1/6th of the proposed capacity is groundwater resources, and to propose going to be a key contributor. plans appropriate to the scale of demand Statement 2 is correct: Steel Authority and aquifer characteristics, and of India Limited (SAIL) is a Maharatna institutional arrangements for company and the largest steel producer participatory management. The NAQUIM and also the highest iron ore miner in programme for mapping of water bearing India. Steel produced by SAIL is used for aquifers has been planned with an aim to railway tracks, defence grade steel for enhance the capacity of states in Ground construction of Arjun battle tank and Water Management and Development. India’s space programme of Chandrayaan and Mangalyaan also use 99. Studying the health and SAIL steel. SAIL supplied around 35400 composition of ‘Benthic Macro- metric tonnes of steel for the construction Invertebrates’ communities is of a of recently inaugurated India’s longest great significance because they bridge - Bogibeel Road-cum-Rail Bridge (a) help in cleaning the oil spill on the river Brahmaputra. This quantity is contamination in ocean little more than 50% of the total quantity waters. of steel supplied for the bridge.

Prelim IAS Test Series (2019) – GS Test 12 (24.02.2019) 48 Geography 2 and Current Affairs Sep, Oct, Nov, Dec 2018 (b) are responsible for algal significance for the nation. This bloom in many polluted water bridge is built over which river? bodies. (a) Narmada (c) are often used as indicators (b) Jhelum of aquatic ecosystem health. (c) Brahmaputra (d) help in mapping the ocean (d) Godavari floor topography with a great accuracy. Answer: (c) Explanation: Answer: (c) The Bogibeel bridge, India’s longest Explanation: road-rail bridge is in the state of Assam. Benthic (meaning “bottom-dwelling”) The bridge, which spans the River macro-invertebrates are small aquatic Brahmaputra between Dibrugarh and animals and the aquatic larval stages of Dhemaji districts of Assam, is of insects. They include dragonfly and immense economic and strategic stonefly larvae, snails, worms, and significance for the nation. This bridge beetles. They lack a backbone, are reduces distances between Assam and visible without the aid of a microscope. Arunachal Pradesh. It would greatly Benthic macro-invertebrates are often enhance "ease of living" in this region. found attached to rocks, vegetation, logs The construction of the bridge will be a and sticks or burrowed into the bottom major boost to improve logistical issues sand and sediments. for the armed forces along the border in Benthic macro-invertebrates are Arunachal Pradesh as India shares a commonly used as indicators of the nearly 4,000 km border with China. biological condition of water bodies. They are reliable indicators because they spend all or most of their lives in water, are easy to collect and differ in their tolerance to pollution. Macro- invertebrates respond to human disturbance in fairly predictable ways, often live for more than a year and, unlike fish, have limited mobility. As they cannot escape pollution, macro- invertebrates have the capacity to integrate the effects of the stressors to which they are exposed. Community characteristics - such as abundance, richness, diversity, evenness, and community composition - can be monitored to determine whether the community is changing over time due to natural or human-caused impacts.

100. Bogibeel bridge, India’s longest road-rail bridge has immense economic and strategic

Prelim IAS Test Series (2019) – GS Test 12 (24.02.2019) 49 Geography 2 and Current Affairs Sep, Oct, Nov, Dec 2018